Anda di halaman 1dari 107

1

CIRUGIA
Masculino de 72 años de edad. Refiere nicturia y polaquiuria durante los últimos cinco años, goteo al final de la micción,
disminución del calibre del chorro urinario. En el último año se agrega dolor testicular y de pene, así como retardo miccional,
seguido por hematuria moderada y retención aguda de orina. Refiere también, lumbalgia progresiva. Al tacto rectal, se aprecia
próstata aumentada de tamaño a expensas de lóbulo derecho con zonas de induración firme, de consistencia pétrea, fija y nodular.
1. Con este cuadro clínico se puede sospechar el diagnóstico de:

a) Adenoma prostático

b) Prostatitis crónica

c) Prostatitis aguda

d) Hipertrofia prostática benigna

e) Carcinoma prostático

2. Los tres signos que se presentan con frecuencia en este padecimiento son:

a) Disuria, polaquiuria y piuria

b) Hematuria, próstata dura al tacto rectal y retención aguda de orina

c) Pérdida de peso, distensión abdominal y retención aguda de orina

d) Dolor lumbar, tenesmo vesical y sedimento urinario

e) Disfunción eréctil, impotencia sexual y priapismo

3. La prueba básica para diagnosticar el estadio clínico de este padecimiento es:

a) Uretrocistografía

b) Colon por enema

c) Rectosigmoidoscopía

d) Tacto rectal

e) Tomografía pélvica

4. El estudio de gabinete específico que debe solicitarse para corroborar el diagnóstico es:

a) Urografía excretora

b) Resonancia magnética nuclear

c) Ultrasonido transrectal

d) Linfografía pélvica
e) Gammagrama óseo

5. El procedimiento terapéutico indicado en este caso es:

a) Resección transuretral de próstata

b) Quimioterapia y radioterapia

c) Prostatectomía, radioterapia, estrogenoterapia

d) Prostatectomía, castración, radioterapia

e) Orquiectomía, quimioterapia, estrogenoterapia

2 CIRUGIA
Femenina de 32 años de edad, divorciada. Refiere cuadros frecuentes de leucorrea amarilla verdosa, con prurito vulvar. Menarca a
los 12 años, IVSA a los 18 años, Gesta: I, Para: I, tres parejas sexuales. Refiere cuadro clínico de siete días, de inicio súbito,
caracterizado por dolor intenso en periné, pulsátil, que se exacerba al caminar, pujar, toser o estornudar; además de fiebre de 38°C.
En la exploración física: quejumbrosa, marcha lenta, temperatura 38.7°C. Abdomen blando, depresible, no doloroso. Se encuentra
masa indurada y dolorosa con eritema suprayacente en forma de herradura en región perianal y tercio posterior de labios mayores.
1. Con base en el cuadro clínico descrito el diagnóstico más probable es:

a) Absceso de la glándula de Bartholin

b) Enfermedad de Bowen

c) Hemorroides internas de tercer grado

d) Infección perianal necrosante

e) Absceso isquiorrectal

2. La causa más frecuente de este padecimiento es:

a) Infección de origen criptoglandular

b) Neoplasia intraepitelial

c) Presión venosa portal aumentada

d) Infección por Neisseria gonorrhoeae

e) Infección por Clostridium perfringens

3. La complicación más frecuente que puede tener esta paciente es:

a) Extensión de la necrosis a vías genitourinarias

b) Formacion de fístulas anorrectales

c) Extensión local de la neoplasia y metástasis


d) Enfermedad hemorroidal aguda

e) Extensión de la infección al aparato genital y cavidad abdominal

4. En este caso, el tratamiento indicado es:

a) Drenaje del absceso en herradura

b) Drenaje del absceso y penicilina procaína

c) Extirpación quirúrgica de la necrosis y tratamiento con oxígeno hiperbárico

d) Escisión, electrodesecación, criocirugía

e) Hemorroidectomía

5. Son factores predisponentes para este padecimiento:

a) Estreñimiento, obesidad, dieta sin fibra vegetal

b) Tratamiento inadecuado o retardado de infección anorrectal primaria

c) Enfermedad de Crohn, trastornos hematológicos y otras inmunodeficiencias

d) Contacto sexual con portadores de Neisseria gonorrhoeae

e) Exposición a carcinógenos, traumatismos y cicatrices

3 CIRUGIA
Masculino de 74 años de edad. Tabaquismo positivo a razón de seis a ocho cigarrillos al día durante los últimos 40 años;
alcoholismo ocasional sin llegar a la embriaguez. Se conoce portador de diabetes mellitus tipo 2 e hipertensión arterial desde hace
15 años. Refiere que desde hace ocho años, presenta episodios repetitivos de dolor en hipogastrio y fosa ilíaca izquierda, el cual
empeora después de comer y se alivia transitoriamente después de defecar. Hace un año, presentó hemorragia rectal indolora y
escasa. Inició el padecimiento actual hace dos días, con dolor abdominal súbito, persistente, localizado en el cuadrante inferior
izquierdo; hiporexia, estado nauseoso y escalofrío. Exploración física: temperatura 38.3°C, TA 140/95 mmHg, FC 96 por minuto,
peso 94 kg, estatura 1.68 m. El abdomen se encuentra distendido, con resistencia muscular involuntaria, hipersensibilidad en el
cuadrante inferior izquierdo, se palpa masa fija y dolorosa. Ruidos intestinales disminuidos.
1. Con base en el cuadro clínico descrito, el diagnóstico más probable es:

a) Diverticulitis

b) Colitis ulcerosa

c) Colitis isquémica

d) Enfermedad de Crohn

e) Hemorroides internas

2. El estudio paraclínico más útil para confirmar el diagnóstico es:

a) Ultrasonografía
b) Proctosigmoidoscopía

c) Tomografía computarizada

d) Radiografía simple de abdomen

e) Estudio radiográfico con contraste de bario

3. El cuadro clínico presentado hace un año muy probablemente se debió a:

a) Angiodisplasia

b) Hemorroides mixtas

c) Hemorragia diverticular

d) Proctitis en enfermedad de Crohn

e) Actividad leve de colitis ulcerosa

4. El tratamiento indicado es:

a) Ayuno, líquidos IV y antibióticos

b) Corticoesteroides y sulfasalazina

c) Inyecciones esclerosantes o fotocoagulación

d) 6-mercaptopurina, sulfasalazina y metronidazol

e) Extirpación quirúrgica del segmento gangrenado

5. Son complicaciones de este padecimiento:

a) Pólipos adenomatosos y neumatosis cistoide del intestino

b) Absceso intraabdominal, fístulas y obstrucción intestinal

c) Prolapso rectal, necrosis cutánea y acrocordones anales

d) Amiloidosis y aumento en la capacidad litogénica de la bilis

e) Angiodisplasia cecal, isquemia fulminante y hemorragia intramural

4 CIRUGIA
Femenina de 26 años de edad, residente en el estado de México, soltera, obrera. Carga genética para diabetes mellitus, tabaquismo
positivo a razón de tres cigarrillos al día durante 10 años; alcoholismo ocasional. Refiere tumoración en cara anterior de cuello, de
seis meses de evolución así como proptosis, angustia, nerviosismo, fatigabilidad, intolerancia al calor, diarrea, pérdida de peso y
palpitaciones. EF: TA 120/80 mmHg, FC 120 por minuto, FR 18 por minuto, temperatura 37°C. Presenta exoftalmos, bocio,
taquicardia rítmica, peristalsis aumentada. Extremidades inferiores con paresia y disminución de masa muscular; edema y
engrosamiento de la piel en regiones pretibiales.
1. De acuerdo con los datos clínicos, la posibilidad diagnóstica en este caso es:
a) Tiroiditis subaguda

b) Tiroiditis de Hashimoto

c) Nódulo tiroideo "caliente"

d) Enfermedad de Graves

e) Hipotiroidismo autoinmunitario

2. La etiología más frecuente de esta enfermedad es:

a) Autoinmunitaria

b) Idiopática

c) Infecciosa

d) Iatrogénica

e) Alteración en el metabolismo del yodo

3. Las características clínicas más frecuentes en esta enfermedad son:

a) Palpitaciones, diarrea, nerviosismo y oftalmopatía

b) Pérdida de peso, taquicardia, intolerancia al calor y diarrea

c) Tirotoxicosis, bocio, oftalmopatía y dermopatía

d) Tirotoxicosis, oftalmopatía, taquicardia y diarrea

e) Tirotoxicosis, bocio, palpitaciones y taquicardia

4. En la variedad subclínica de este padecimiento el trastorno del ritmo cardiaco más frecuente es:

a) Extrasistolia supraventricular

b) Extrasistolia ventricular

c) Fibrilación auricular

d) Fibrilación ventricular

e) Bloqueo auriculo-ventricular

5. La complicación más grave de este padecimiento es:

a) Mixedema

b) Coma mixedematoso
c) Tormenta tiroidea

d) Tetania por hipocalcemia

e) Fibrilación auricular

5 CIRUGIA
Femenina de 35 años de edad, originaria del estado de Hidalgo, con malos hábitos higiénico-dietéticos y fecalismo al aire libre.
Tabaquismo negativo, alcoholismo ocasional dos veces al mes al ingerir medio litro de pulque, cuadros diarreicos cada semana
con duración de dos a tres días, tratada con herbolaria. Padecimiento actual: inició un mes antes de su ingreso con escalosfríos y
sudoración. Posteriormente presentó hipertermia cuantificada en 39.6°C y una semana antes de su ingreso, dolor en hemitórax
región subcostal derecha con tos seca en accesos, ataque al estado general y pérdida de peso cuantificada en 7 kg. A la exploración
física: talla 1.55 m, peso 50 kg, TA 110/70 mmHg, FC 98 por minuto, temperatura 38.9°C, adelgazada, tinte subictérico,
rubicundez. Precordio con ruidos cardiacos rítmicos, taquicardia, sin otros fenómenos agregados. Abdomen con hepatomegalia
dolorosa, cuatro centímetros por debajo del borde costal. No esplenomegalia. Peristaltismo normal, sin datos de irritación
peritoneal.
1. El estudio de imagen indicado para el diagnóstico en esta paciente es:

a) Telerradiografía de tórax

b) Placa simple de abdomen en dos posiciones

c) Angiografía mesentérica

d) TAC de abdomen

e) Resonancia magnética abdominal

2. La prueba de laboratorio para confirmar el diagnóstico en este caso es:

a) Hemaglutinación indirecta

b) Biometría hemática completa

c) Fosfatasa alcalina

d) Aminotransferasas

e) Bilirrubinas

3. La vía de entrada del agente agresor al hígado es:

a) Linfática

b) Arteria hepática

c) Sistema venoso portal

d) Sistema venoso caval

e) Contigüidad

4. La localización más frecuente de este padecimiento es el lóbulo hepático:


a) Izquierdo

b) Derecho anterosuperior

c) Derecho anteroinferior

d) Cuadrado

e) Riedel

5. El tratamiento de elección en este caso es:

a) Diyodohidroxiquinoleína 600 mg tres veces al día vía oral

b) Metronidazol 500 mg tres veces al día vía intravenosa

c) Metronidazol 250 mg tres veces al día vía intravenosa

d) Metronidazol 50 mg tres veces al día vía intravenosa

e) Itraconazol 100 mg una vez al día vía oral

6 CIRUGIA
Masculino de 65 años de edad. Presenta antecedentes de importancia de tabaquismo positivo por 30 años a razón de 40 cigarrillos
al día; hipertenso y con historia clínica de diabetes mellitus tipo 2, desde hace diez años. Acude a consulta por presentar dolor
epigástrico tipo cólico, intenso, irradiado hacia hipocondrio izquierdo; ataque al estado general, astenia, adinamia y pérdida de
peso de aproximadamente 20 kg en tres meses. A la exploración física: TA 130/90 mmHg, FC 90 por minuto, FR 19 por minuto,
temperatura 37.0°C. Sólo se detecta dolor a la palpación profunda en epigastrio, sin otros datos de importancia.
1. Ante este cuadro clínico, el diagnóstico de elección es:

a) Enfermedad ácido péptica

b) Colitis amibiana

c) Cuadro doloroso abdominal

d) Síndrome anémico

e) Síndrome de consunción

2. La causa de esta entidad es:

a) Absceso hepático

b) Úlcera péptica

c) Hernia hiatal

d) Cáncer gástrico

e) Colecistitis crónica
3. El estudio de gabinete de elección es:

a) Ultrasonido de hígado y vías biliares

b) Serie esofagogastroduodenal

c) Radiografía simple de abdomen

d) Gammagrama de hígado

e) Colecistografía oral

4. Una enfermedad que juega papel preponderante como factor de riesgo para este padecimiento es:

a) Colitis amibiana

b) Úlceras agudas de estómago

c) Gastritis crónica atrófica

d) Úlcera péptica benigna

e) Litiasis vesicular

5. El estudio paraclínico que permite un diagnóstico temprano en este padecimiento es:

a) Endoscopía digestiva con toma de biopsia

b) Gammagrama de hígado

c) Colecistografía oral

d) Colangiografía transduodenal

e) Biopsia por aspiración con aguja fina

7 CIRUGIA
Femenina de 40 años de edad, con antecedente de pirosis y regurgitación durante los últimos cinco años. Esta sintomatología se ha
hecho más frecuente recientemente, sobre todo al agacharse y con predominio nocturno, además refiere que se ha agregado tos
matutina y dolor punzante localizado en epigastrio e irradiado hacia región precordial.
1. Con la información anterior, el diagnóstico más probable es:

a) Gastritis crónica

b) Úlcera gástrica

c) Acalasia

d) Reflujo gastroesofágico

e) Divertículo esofágico

2. El estudio indicado para corroborar su diagnóstico es:


a) Manometría esofágica

b) pHmetría

c) Serie esofagogastroduodenal

d) Panendoscopia

e) Toma de biopsia

3. La complicación más frecuente de esta alteración es:

a) Esofagitis

b) Hemorragia

c) Estenosis

d) Perforación

e) Malignización

4. De acuerdo con la sintomatología, ¿con qué entidad se debe hacer el diagnóstico diferencial?

a) Osteocondritis

b) Angina de pecho

c) Pancreatitis

d) Colecistitis

e) Hernia paraesofágica

5. El tipo de hernia hiatal más común es:

a) La diafragmática

b) La postraumática

c) La paraesofágica

d) Por esófago corto

e) Por deslizamiento

8 GINECOOBSTETRICIA
Paciente de sexo femenino de 32 años de edad. IVSA a los 24 años, G: II, C: I, hace dos años por desproporción cefalopélvica, sin
complicaciones. Actualmente con embarazo de 32 semanas por FUM, normoevolutivo. Acude a urgencias con cuadro de un día de
evolución caracterizado por fiebre no cuantificada, escalofríos, náusea, dolor en región costolumbar y actividad uterina irregular.
A la exploración física: diaforética, TA 110/60 mmHg, FC 98 por minuto, FR 26 por minuto, temperatura 39°C. Abdomen con
fondo uterino a 31 cm del borde superior del pubis. FCF 164 por minuto. Giordano bilateral positivo. Tacto vaginal: orificio
cervical externo dehiscente.
1. El cambio fisiológico en el tracto urinario que favorece esta entidad es:

a) El pH urinario es bajo

b) Disminución de la filtración glomerular

c) Dilatación de los ureteros y pelvis renal

d) Aumento del tono vesical

e) Aumento de la peristalsis ureteral

2. Con los datos clínicos su posibilidad diagnóstica es:

a) Pielonefritis

b) Uretrotrigonitis

c) Cistitis

d) Bacteriuria asintomática

e) Glomerulonefritis

3. En caso de aislarse bacterias, las que con mayor frecuencia causan esta entidad son:

a) E. coli y Klebsiella sp

b) E. coli y Enterococcus sp

c) Klebsiella y Proteus sp

d) Enterococcus sp y Streptococcus del grupo beta

e) Staphylococcus saprophyticus y Streptococcus sp

4. Esta entidad se caracteriza por:

a) Ser el padecimiento urinario más frecuente durante el embarazo

b) Ser más frecuente después del segundo trimestre y unilateral en más de la mitad de los casos

c) Contar con urocultivo con más de 100 000 microorganismos por mL de varias bacterias

d) Presencia de hematuria

e) Fiebre alta y continua

5. La conducta a seguir en este caso consiste en:

a) Tratamiento ambulatorio con acidificantes urinarios


b) Tratamiento ambulatorio con antibióticos

c) Solicitar urocultivo y esperar resultado

d) Hospitalización para iniciar antibiótico vía parenteral

e) Hospitalización para inducir madurez pulmonar e interrumpir embarazo

9 GINECOOBSTETRICIA
Paciente de sexo femenino de 32 años de edad. Antecedente de tabaquismo positivo a razón de dos cigarrillos al día durante seis
años. IVSA a los 24 años, monógama, G: IV, C: I, A: I, P: I, con embarazo de 34.2 semanas por FUM, sin antecedentes de control
prenatal. Acude a urgencias por hemorragia transvaginal de color rojo rutilante, en cantidad semejante a una menstruación, de dos
horas de evolución, no relacionada con actividad física, niega actividad uterina. Exploración física: TA 120/80 mmHg, FC 80 por
minuto, FR 20 por minuto, temperatura 37°C; fondo uterino palpable a 31 cm del borde superior del pubis, tono uterino normal,
FCF 150 por minuto; producto en situación longitudinal y presentación cefálica. Se solicita USG obstétrico que informa:
embarazo de 34 semanas con borde placentario en el segmento inferior a cinco centímetros del orificio cervical interno, líquido
amniótico sin modificaciones.
1. Con base en los hallazgos, el diagnóstico más probable es:

a) Placenta previa de inserción baja

b) Placenta previa central total

c) Placenta previa marginal

d) Vasa previa

e) Desprendimiento prematuro de placenta normoinserta

2. Son características de esta entidad:

a) El útero se encuentra hipertónico, acompañado de dolor

b) El riesgo de recurrencia es 2 veces menor que la población general

c) La hemorragia es color rojo rutilante, no acompañada de dolor

d) El tabaquismo no se relaciona con este padecimiento

e) Ocurre después de la semana 20 de gestación, la hemorragia es color rojo oscuro y acompañada de dolor

3. Son factores de riesgo para esta entidad, EXCEPTO:

a) Multiparidad

b) Primigestas

c) Cesáreas previas

d) Antecedente de legrados

e) Embarazo gemelar
4. El tratamiento de elección de acuerdo con el diagnóstico inicial y la edad gestacional es:

a) Inductoconducción de trabajo de parto

b) Útero-inhibición

c) Cesárea

d) Observación y reposo en decúbito lateral

e) Histerectomía en bloque

5. La complicación que se puede esperar en este caso es:

a) Acretismo placentario

b) Dehiscencia de histerorrafia

c) Ruptura prematura de membranas

d) Inversión uterina

e) Coagulopatía por consumo

10 GINECOOBSTETRICIA
Paciente de sexo femenino de 38 años de edad. Antecedente de hipertensión arterial crónica en tratamiento. G: IV, P: III. Cursa
con embarazo de 34 semanas por FUM, clínica y ultrasonidos del primero y segundo trimestres. Tabaquismo de 10 cigarrillos al
día durante el embarazo. Refiere dolor tipo cólico en región pélvica y lumbar, así como actividad uterina esporádica y
hemorragia transvaginal en cantidad semejante a una menstruación, de color rojo oscuro. Exploración física: TA 140/90 mmHg,
FC 90 por minuto, fondo uterino a 32 cm del borde superior del pubis, FCF apenas perceptible con Pinard, de 120 por minuto,
útero hipertónico. Especuloscopía: hemorragia transcervical escasa, constante, de color rojo oscuro, cérvix con dos centímetros
de dilatación.
1. Esta entidad se considera posible en el período comprendido:

a) Después de la semana 20 y 2 horas antes del inicio del trabajo de parto

b) Después de la semana 20 y antes del tercer período del trabajo de parto

c) Sólo en el tercer trimestre

d) Sólo durante el trabajo de parto

e) Sólo en la tercera etapa del trabajo de parto

2. El factor de riesgo más importante para la aparición de esta entidad es:

a) Tabaquismo

b) Edad materna

c) Edad gestacional

d) Antecedente de tres partos


e) Hipertensión arterial

3. Son exámenes de laboratorio que solicita en este caso:

a) Plaquetas, TP, TTP, formación y retracción del coágulo

b) Examen general de orina, citogenético y cultivo celular del líquido amniótico

c) Biometría hemática, tiempo de desintegración del coágulo y espectrofotometría del líquido amniótico

d) Biometría hemática, química sanguínea y prueba de Clements

e) Transaminasas, deshidrogenasa láctica, plaquetas y examen general de orina

4. En esta paciente, con feto vivo e hipertonía uterina, el tratamiento de elección es:

a) Reposo, hidratación, vigilancia intrahospitalaria hasta las 36 semanas de embarazo

b) Inductoconducción del trabajo de parto con oxitocina

c) Inductoconducción del trabajo de parto con prostaglandinas

d) Cesárea inmediata

e) Perfil biofísico y manejar de acuerdo con el resultado

5. ¿Qué complicación materna se puede presentar en este caso?

a) Ruptura uterina

b) Coagulopatía por consumo

c) Acretismo placentario

d) Insuficiencia cardíaca congestiva

e) Inversión uterina

11 GINECOOBSTETRICIA
Femenina de 28 años de edad. AGO: IVSA a los 26 años, G: I, embarazo actual de 38 semanas por FUM. Refiere actividad
uterina regular. Exploración física: talla 1.55 m, peso 66 kg, abdomen con fondo uterino de 31 cm, PUV longitudinal, cefálico,
con FCF 142 por minuto, se detecta actividad uterina regular. Tacto vaginal: cérvix blando, central, semiborrado, con tres
centímetros de dilatación y membranas íntegras. A su ingreso, se realiza valoración de pelvis ósea.
1. El parámetro que se debe conocer para valorar el estrecho medio es:

a) Diámetro transverso o interespinoso

b) Tuberosidades isquiáticas

c) La dirección del cóccix

d) La abertura del ángulo subpúbico


e) La prominencia de la quinta vértebra sacra

2. La medida normal del estrecho medio (en centímetros) debe ser:

a) Menor de 8

b) 8

c) 9

d) 9.5

e) Más de 10

3. El diámetro que se puede medir clínicamente en el estrecho superior es:

a) Transverso

b) Conjugado verdadero

c) Conjugado diagonal

d) Sagital posterior

e) Oblicuo

4. Para considerarse normal, dicho diámetro debe medir (en centímetros):

a) 9.5

b) 10.5

c) 11.5

d) 12.5

e) 13.5

5. El diámetro más corto del estrecho superior es:

a) Conjugado verdadero

b) Conjugado obstétrico

c) Conjugado diagonal

d) Transverso

e) Oblicuo

12 GINECOOBSTETRICIA
Paciente de sexo femenino de 24 años de edad, con antecedente heredofamiliar de madre y una hermana con diabetes mellitus
tipo 2. Menarca a los 12 años, IVSA a los 19 años, G: II, C: I, por desproporción cefalopélvica, producto con peso al nacer de 3
900 g. Actualmente, cursa con embarazo de 28 semanas por FUM y control prenatal irregular. A la exploración física: signos
vitales normales. Abdomen con fondo uterino a 26 cm del borde suprapúbico. PUV cefálico, longitudinal, dorso a la derecha,
FCF de 146 por minuto. Glucemia sérica en ayuno de 120 mg/dL.
1. Se recomienda descartar diabetes gestacional en pacientes con las siguientes características:

a) Obesidad importante (IMC>30) y antecedente de patología obstétrica

b) Tabaquismo y antecedentes familiares de DM en primer grado

c) Hipertensión arterial y antecedente personal de diabetes gestacional

d) Glucosuria y edad menor de 25 años

e) Microalbuminuria y antecedente familiar de DM

2. La prueba de detección o tamiz de glucosa se realiza preferentemente entre las semanas:

a) 12 a 15

b) 16 a 19

c) 20 a 23

d) 24 a 28

e) 29 a 32

3. Esta prueba se considera positiva si se encuentra una cifra mayor de (en mg/dL):

a) 120

b) 130

c) 140

d) 150

e) 160

La prueba de tamiz de glucosa informa una cifra de 153 mg/dL.


4. Con base en el informe anterior se indica:

a) Repetir la prueba en dos semanas

b) Curva de tolerancia oral a la glucosa

c) Determinación de hemoglobina glucosilada (HbA1C)

d) Determinación de glucosa sérica al azar

e) Manejo con insulina 0.3 UI por kg de peso ideal sin embarazo

5. La prueba de tamiz se considera diagnóstica de diabetes mellitus si se encuentra una cifra mayor de (en mg/dL):
a) 140

b) 150

c) 160

d) 170

e) 180

13 GINECOOBSTETRICIA
Mujer de 52 años de edad. Acude a consulta por ausencia de ciclos menstruales desde hace seis meses. Presenta labilidad
emocional, artralgias, oleadas de calor (bochornos) y resequedad vaginal. Se inicia terapia hormonal con estrógenos.

1. ¿Qué estudio solicita para corroborar su diagnóstico?

a) Hormona estimulante del tiroides (TSH)

b) Triyodotironina(T3) y tiroxina (T4)

c) Cuantificación de cetoesteroides

d) Cuantificación de cortisol

e) Hormona folículo estimulante (FSH) y luteinizante (LH)

2. En este estudio, espera encontrar:

a) FSH y LH aumentadas

b) TSH aumentada

c) Cetoesteroides aumentados

d) Cortisol elevado

e) T3 y T4 elevadas

3. El efecto cardioprotector de los estrógenos, se debe al aumento de:

a) Colesterol

b) Lipoproteínas de muy baja densidad

c) Lipoproteínas de baja densidad

d) Lipoproteínas de intermedia densidad

e) Lipoproteínas de alta densidad

4. Es una contraindicación absoluta para iniciar terapia con estrógenos:


a) Miomatosis uterina

b) Hipertensión aterial

c) Antecedente de tromboembolismo

d) Diabetes mellitus

e) Mastopatía fibroquística

5. ¿Qué otro estudio es importante solicitar?

a) Perfil de lípidos

b) USG de mama

c) Mastografía

d) Densitometría ósea

e) USG pélvico

14 GINECOOBSTETRICIA
Femenina de 35 años de edad. Acude a urgencias por presentar tumoración en labio mayor derecho de 2 días de evolución, con
dolor, dificultad para deambular y sentarse. A la exploración física: temperatura de 37.8°C. Se observa aumento de volumen de
labio mayor derecho con tumoración de 6x5 cm, hiperémica, dolorosa a la palpación, consistencia blanda. Al tacto vaginal, útero
y ovarios sin alteraciones.
1. Con los hallazgos clínicos, la principal posibilidad diagnóstica es:

a) Lipoma vulvar

b) Furunculosis

c) Bartholinitis

d) Quiste endometriósico vulvar

e) Quiste de la glándula de Bartholin

2. El principal agente etiológico involucrado en esta entidad es:

a) Estreptococos

b) Neisseria gonorrhoeae

c) Virus del papiloma humano

d) Estafilococos

e) Proteus

3. Es correcto en esta entidad que:


a) Están implicados cambios hormonales

b) Habitualmente es unilateral

c) Existen ciclos anovulatorios

d) Es más frecuente en mujeres con hipertricosis

e) Es exclusivo de mujeres en edad fértil

4. El manejo inicial en este caso consiste en:

a) Bartholinectomía

b) Alta a su domicilio con antibiótico

c) Hospitalización e impregnación con antibióticos

d) Drenaje de la lesión y antibiótico

e) Exéresis de la lesión

5. En caso de recidiva, el manejo será:

a) Iniciar doble esquema de antibiótico

b) Marsupialización

c) Bartholinectomía

d) Antibiótico oral y local

e) Tomar biopsia de la lesión

15 GINECOOBSTETRICIA
Femenina de 49 años de edad. Tiene diagnóstico de mastopatía fibroquística desde los 26 años, sin tratamiento específico.
Diabética tipo 2 desde hace 12 años, tratada con hipoglucemiantes orales. Tabaquismo positivo a razón de 10 cigarrillos al día
durante 15 años. AGO: menarca a los 11 años, IVSA a los 20 años, dos parejas sexuales; utilizó anticonceptivos orales por
espacio de ocho años; ciclos regulares 28/4, G:0. Hace dos meses se detecta en mama derecha tumoración de tres centímetros de
diámetro, no dolorosa. A la exploración física: TA 140/ 95 mmHg,FC 80 por min, FR 18 por min, T 36°C. Obesa grado II.
Mamas simétricas sin alteraciones en su superficie, se corrobora tumoración dura, fija, adherida a planos superficial y profundo,
no se observa secreción a través de pezón ni adenopatías.
1. Son factores de riesgo para esta entidad:

a) Nuliparidad, menopausia tardía y antecedentes familiares en línea directa

b) Diabetes mellitus, enfermedad fibroquística y obesidad

c) Antecedente familiar en línea directa, hipertensión arterial y nuliparidad

d) Antecedente familiar en línea directa, tabaquismo y menopausia tardía


e) Menopausia tardía, múltiples parejas sexuales y diabetes mellitus

2. La localización más frecuente de esta entidad es en:

a) Pezón

b) Retroareolar

c) Cuadrante inferior externo

d) Cuadrante superior interno

e) Cuadrante superior externo

3. El sitio de metástasis más frecuente de esta neoplasia es:

a) Bazo

b) Hígado

c) Pulmón

d) Cerebro

e) Hueso

4. Condición que ejerce efecto protector contra la aparición de esta neoplasia:

a) Ooforectomía

b) Obesidad

c) Nuliparidad

d) Terapia hormonal de remplazo

e) Menarca tardía

5. La variedad histológica más frecuente en el cáncer de mama es:

a) Ductal infiltrante

b) Intraductal no infiltrante

c) Lobulillar infiltrante

d) Mucinoso

e) Medular

16 MEDICINA FAMILIAR
Femenina de 42 años de edad. Es profesionista, casada y madre de dos hijos. Antecedente de histerectomía por miomatosis sin
salpingooforectomía. En fecha reciente tuvo conflictos con su pareja por sospechar infidelidad. Ha faltado frecuentemente al
trabajo y ha descuidado a sus hijos. Presenta diversos datos de somatización e ideas suicidas. Acude a consulta por cuadro de un
año de evolución con aumento de peso, anhedonia, tristeza patológica, desesperanza, frustración, aislamiento social e
hipoactividad. A la exploración física: talla 1.66 m, peso 78 k, TA 100/70 mmHg, poco expresiva, con desánimo, hiperlordosis
dorsal, tendencia al llanto durante la entrevista y se frota las manos con frecuencia.
1. Es característica de la tristeza patológica que presenta la paciente:

a) Ser una estimulación del sentimiento

b) Aparecer sin motivo fácilmente identificable

c) Ser coherente con lo que la desencadena

d) Una intensidad y duración adecuadas a la situación desencadenante

e) No afectar el funcionamiento global de manera importante.

2. El término anhedonia se refiere a:

a) Pérdida de la sensación de placer, especialmente en la esfera sexual

b) Estado afectivo displacentero que suele acompañarse de palidez, disnea y taquicardia.

c) Temor ante un peligro real o imaginario

d) Logorrea o verborrea

e) Falta de capacidad o potencia para el trabajo

3. Con base en lo anterior el diagnóstico de la paciente es:

a) Trastorno bipolar

b) Trastorno de ansiedad

c) Depresión mayor

d) Depresión atípica

e) Episodio maníaco

4. Es característico de esta entidad:

a) Ser más frecuente en la quinta década de la vida

b) Ser más frecuente en hombres

c) Tiende a la cronicidad con recaídas

d) Remisión completa entre los episodios y recuperación funcional premórbida

e) Asociación con disfunción tiroidea

5. Indica a su paciente tratamiento con amitriptilina. Este fármaco se caracteriza por:


a) Ser un inhibidor de la aminooxidasa

b) Tener como efectos colaterales sequedad de boca, náusea y vómito

c) Aumentar los niveles de dopamina e histamina cerebrales

d) Responder al tratamiento desde la primera semana

e) Causar como efecto secundario manchas hipocrómicas y prurito en piel

17 MEDICINA FAMILIAR
Niña de seis meses de edad. A partir del día de hoy, en forma súbita, presenta cuatro evacuaciones liquidas sin moco ni sangre,
náusea y vómito de contenido alimentario en dos ocasiones; aumento de la temperatura corporal de 37.8°C; su madre la refiere
irritable y sedienta. Exploración física: peso 6 600 g, talla 69 cm, temperatura 38°C, inquieta, irritable, taquipneica, palidez de
tegumentos +, mucosa oral con regular estado de hidratación, fontanela hipotensa y globos oculares con lágrimas escasas; piel
poco turgente, llenado capilar de dos segundos. Sodio sérico de 132 mEq/L. Se indica plan B con vida suero oral.
1. El agente etiológico más frecuentemente implicado en este padecimiento es:

a) Shigella flexneri

b) Escherichia coli enteropatógena

c) Campylobacter jejuni

d) Rotavirus

e) Adenovirus

2. Con base en la concentración de sodio sérico la clase de deshidratación que presenta la paciente es:

a) Moderada de 2° grado

b) Intensa de 3er grado

c) Hiponatrémica

d) Isonatrémica

e) Hipernatremica

3. Al disolver el sobre en un litro de agua la osmolaridad total es de (mmol/L):

a) 261

b) 284

c) 311

d) 324

e) 341
4. La dosis de la preparación de acuerdo al plan indicado en esta paciente es (mL/Kg peso):

a) 50

b) 75

c) 100

d) 125

e) 150

5. La duración en horas del plan B de hidratación oral indicado en esta paciente es de:

a) 1

b) 2

c) 3

d) 4

e) 5

18 MEDICINA FAMILIAR
Femenina de 22 años de edad. Originaria del D.F., taxista. Antecedentes de obesidad y diabetes mellitus tipo 2 en madre y una
hermana. Tabaquismo positivo a razòn de seis cigarrillos al día durante cinco años. Acude a consulta por lumbalgia crónica
agudizada. Se le ha indicado dieta de reducción de peso, diversos analgésicos y ejercicios de fortalecimiento lumbar. Muestra
poco apego a las indicaciones. Se indicó metamizol IM. En la radiografía de columna se observa hiperlordosis lumbar.
Exploración física: TA 130/90 mmHg, talla 1.55 m, peso 78 kg, tegumentos de coloración adecuada, ausencia de hirsutismo, sin
tumoraciones en cuello. Abundante panículo adiposo abdominal. Perímetro de cadera 101 cm y de cintura 88 cm. Presencia de
contractura muscular paravertebral. Sensibilidad y tono muscular normales en miembros inferiores.
1. El índice de masa corporal (IMC) de esta paciente es (en kg/m2):

a) 30.5

b) 31.5

c) 32.5

d) 33.5

e) 34.5

2. Se considera obesidad de acuerdo a la norma oficial mexicana (NOM), cuando el IMC (en kg/m2) es mayor de:

a) 24

b) 27

c) 29

d) 31
e) 33

3. El criterio para detectar obesidad, evaluando la distribución de tejido adiposo es:

a) Índice corporal recíproco

b) Índice de Quetelet

c) Índice intermembral

d) Índice cintura-cadera

e) Circunferencia de cintura

4. El perímetro de la cintura se mide a nivel de:

a) La última costilla flotante

b) El borde costal (10a costilla)

c) Cicatriz umbilical

d) Dos centímetros por debajo de la cicatriz umbilical

e) Espina iliaca anterosuperior

5. El índice cintura-cadera que presenta esta paciente es:

a) 0.47

b) 0.57

c) 0.67

d) 0.77

e) 0.87

19 MEDICINA FAMILIAR
Masculino de 23 meses de edad. Nació por vía vaginal con peso de tres kilogramos y talla 50 cm. Actualmente camina sin ayuda,
su lenguaje es en pequeñas frases, controla esfínteres y está incorporado a la alimentación familiar. Esquema de vacunación:
tiene una dosis de pentavalente, tres de Sabin y una de BCG. Peso actual de 12 kg. Es llevado a control a su clínica.
1. A esta edad el niño se considera como:

a) Maternal

b) Lactante menor

c) Lactante mayor

d) Preescolar
e) Escolar

2. El crecimiento ponderal a esta edad es aproximadamente de:

a) 750 g/mes

b) 1000 g/año

c) 2000 g/año

d) 3000 g/año

e) 4000 g/año

3. La talla promedio que se alcanza a la edad de dos años es (en centímetros) de:

a) 75

b) 86

c) 89.5

d) 97.5

e) 100

4. El desarrollo tiene momentos particulares en que se obtienen máximos logros en crecimiento; la etapa en la que se encuentra
nuestro paciente es de tipo:

a) Genital

b) Equilibrio

c) Linfático

d) General

e) Neural

5. De acuerdo con su desarrollo, este paciente:

a) Corre torpemente, sube escaleras y come solo

b) Corre bien, sube y baja escaleras y come con cuchara

c) Camina con apoyo y atrapa una pelota con las dos manos

d) Brinca alternando ambos pies y sube escaleras

e) Cuenta monedas y apila torres de ocho cubos

20 MEDICINA FAMILIAR
Masculino de nueve meses de edad. Es traído a consulta por su mamá para control del niño sano. Antecedentes: producto de
gestación de 40 semanas, Apgar de 8/9, peso al nacimiento 2 750 g, talla 49 cm, valoración de Capurro compatible con edad
gestacional, tamiz metabólico normal. Alimentado con leche materna, ablactado a los cuatro meses con jugo de frutas y papillas.
Inmunizaciones al corriente.
1. De acuerdo con la edad, ¿cuál es el tipo de leche que debe recibir?

a) Industrializada entera

b) Industrializada maternizada con hierro

c) Industrializada semidescremada

d) Industrializada maternizada

e) Materna

2. Para esta edad, ¿cuál es el peso corporal esperado? (en gramos):

a) 7 250

b) 7 500

c) 7 750

d) 8 000

e) 8 250

3. La talla ideal (en cm) debe ser:

a) 60

b) 65

c) 69

d) 79

e) 89

4. Dentro del programa de ablactación de este niño, ¿qué alimento debe incluirse?

a) Carne de pollo

b) Carne de res

c) Yema de huevo

d) Cereales mixtos

e) Pescados

5. Al valorar el desarrollo neurológico, este niño debe:

a) Gatear e incorporarse con ayuda


b) Incorporarse sin ayuda

c) Caminar con ayuda

d) Gatear torpemente

e) Mantenerse sentado

21 MEDICINA FAMILIAR
Mujer de 25 años de edad. Menarca a los 13 años, IVSA a los 18, ha tenido varios compañeros sexuales no circuncidados. G: I,
P: I. Utiliza anticonceptivos orales. Acude a consulta por presentar, desde hace dos meses, leucorrea profusa, gris verdosa, de
aspecto espumoso y fétida. Disuria, dispareunia y prurito con ardor vulvovaginal que se exacerba con la menstruación y después
de ésta.
1. Con base en el cuadro clínico, el diagnóstico más probable es:

a) Vaginosis bacteriana

b) Herpes genital

c) Gonorrea

d) Tricomoniasis

e) Infección por clamidia

2. Es una característica de esta entidad:

a) No se debe dar tratamiento a la pareja

b) No se ha relacionado a complicaciones perinatales

c) Siempre causa sintomatología especialmente en el hombre

d) Trichomonas vaginalis puede adherirse al epitelio vaginal por medio de adhesinas

e) El tratamiento con clotrimazol crema es altamente efectivo (98%)

3. Se realiza especuloscopía. Esperaría encontrar:

a) Secreción amarillo-verdosa y puntilleo hemorrágico en cérvix

b) Secreción grisácea con olor aminado

c) Leucorrea abundante procedente de canal endocervical

d) Presencia de ulceraciones con halo hiperémico

e) Lesión papuloerosiva de bordes bien definidos, indurada de fondo limpio

4. El estudio que solicita para apoyar su diagnóstico es:

a) Reacción en cadena de polimerasa


b) Antígenos treponémicos absorbidos fluorescentes

c) Cultivo en medio de Thayer-Martin

d) Examen en fresco con solución salina

e) Prueba con técnica de ELISA

5. El tratamiento de elección en este caso es:

a) Aciclovir

b) Doxiciclina

c) Penicilina benzatínica

d) Espectinomicina

e) Metronidazol

22 PEDIATRIA
Femenina de 12 años de edad. Hace seis meses presentó varicela. Desde hace dos semanas presenta lesiones eritematosas en la
piel cabelluda y prurito local, los cabellos se desprenden dejando un aspecto punteado negro. Exploración física: se aprecian seis
placas redondas de 0.5 a 1.5 cm, con presencia de escamas, se observan cabellos cortos y quebradizos, no se aprecian
adenopatías, el resto de la exploración es normal.
1. Con los datos anteriores, el diagnóstico que emite es:

a) Tinea versicolor

b) Alopecia areata

c) Tinea capitis

d) Tricotilomanía

e) Querión

2. El agente etiológico más frecuente en este padecimiento es:

a) Malassezia furfur

b) Candida albicans

c) Trichophyton tonsuras

d) Trichophyton schoenleinii

e) Trichophyton concentricum

3. El examen que confirma su diagnósticos es:

a) Cultivo en medio Sabouraud


b) Micológico de cabellos íntegros que ceden fácilmente

c) Biopsia de piel cabelluda

d) Microscópico directo de cabellos infectados con KOH

e) Exclusivamente por clínica

4. Usted considera que este padecimiento:

a) Afecta frecuentemente en edades adultas

b) Es contagioso, de predominio en etapas escolares

c) No es contagioso

d) Es más frecuente en individuos inmunocomprometidos

e) Cuando cura, deja una alopecia residual

5. El tratamiento para esta paciente es:

a) Sulfuro de selenio

b) Ketoconazol tópico

c) Clotrimazol tópico

d) Lindano

e) Ketoconazol oral

23 MEDICINA FAMILIAR
Masculino de 75 años de edad. Toma antihipertensivos desde hace 15 años. Consulta por dermatosis localizada en hemicara
derecha con eritema, vesículas abundantes, costras melicéricas y hemáticas de tres días de evolución. Una semana antes inició
con sensación de quemadura. Utilizó remedios caseros sin obtener mejoría.
1. Ante este cuadro clínico el diagnóstico más probable es:

a) Herpes zoster

b) Dermatitis atópica

c) Pénfigo vulgar

d) Impétigo

e) Erisipela

2. La etiología de esta enfermedad es:

a) Ingestión crónica de antihipertensivos


b) Virus herpes simple tipo I

c) Virus varicela zoster

d) Estreptococo beta hemolítico grupo A

e) Desconocida

3. Las estructuras que afecta la enfermedad que diagnosticó son piel y:

a) Músculo

b) Tejido celular subcutáneo

c) Nervios periféricos

d) Capilares sanguíneos

e) Sistema nervioso central

4. El tratamiento de elección ante un caso con tres días de evolución como éste es:

a) Fomentos con antisépticos

b) Corticoesteroides tópicos

c) Corticoesteroides sistémicos

d) Antivirales

e) Queratolíticos

5. Una complicación frecuente de esta enfermedad es:

a) Pérdida de la fonación

b) Dolor crónico

c) Incapacidad para la deglución

d) Cicatriz queloide

e) Hipopigmentación

24 MEDICINA FAMILIAR
Masculino de 59 años de edad. Originario del estado de México. Fuma seis cigarrillos al día desde hace 20 años. Ingiere bebidas
alcohólicas una vez a la semana sin llegar a la embriaguez. Desde hace tres años presenta polaquiuria, nicturia y micción
imperiosa. Hace un mes se agrega disminución del calibre y fuerza del chorro, micción prolongada y goteo postmiccional.
Exploración física: TA 135/90 mmHg, peso 80 kg, talla 1.70 m, tacto rectal con próstata crecida, uniforme, de consistencia
elástica con la superficie lisa, sin presencia de globo vesical. Reflejos cremastérico, anal superficial y bulbocavernoso normales.
Se solicita EGO, urocultivo, química sanguínea y antígeno prostático específico (PSA. Éste último informa 5 ng/dL.
1. El valor considerado como normal del antígeno prostático específico (PSA en ng/mL) es:
a) 0 a 4

b) 2 a 6

c) 4 a 8

d) 6 a 10

e) > 10

Se determina el cociente PSA libre / PSA total


2. El cociente PSA libre y total considerado como normal es:

a) 0.5 a 0.9

b) 0.10 a 0.14

c) 0.15 a 0.25

d) 0.26 a 0.35

e) 0.36 a 0.40

3. Es un fármaco que se utiliza para el tratamiento médico de este padecimiento:

a) Hidroclorotiacida

b) Alfametildopa

c) Diclofenaco

d) Doxazocina

e) Fluoxetina

4. Un efecto secundario frecuente por el uso de este fármaco es:

a) Edema

b) Rash mucocutáneo

c) Orina rojiza

d) Evacuaciones disminuidas de consistencia

e) Hipotensión arterial

5. Otros efectos secundarios que se presentan con el uso de este fármaco son:

a) Mareos, astenia, ortostatismo y somnolencia

b) Visión borrosa, vértigo y nistagmus


c) Dolor abdominal y síndrome anémico

d) Estreñimiento y nàusea

e) Ortostatismo, diplopia y tinnitus

25 MEDICINA INTERNA
Femenina de 37 años de edad. Antecedente de diabetes mellitus en la familia. G: II, P: 0, A: II. FUM hace diez días. Solicita
atención médica por debilidad, cefalea occipital, mareos, palpitaciones y disnea. A la Exploraciòn fìsica, TA 90/50 mmHg, pulso
120 por minuto, temperatura 37.8°C, FR 32 por minuto. Se encuentra consciente y pálida, tiene dificultad para responder al
interrogatorio. Soplo en mesocardio, holosistólico suave (2 de 6)Abdomen doloroso a la palpación en hipogastrio. BH con 12
300 leucocitos y 2.5 millones de glóbulos rojos por mm3, Hb 5.6 g/dL, VCM 68 mm3, CMHG 23 g/dL, Urea 59 mg/dL,
Creatinina 1.5 mg/dL.
1. Es probable que la anemia en este caso se deba a:

a) Incremento en la utilización de hierro

b) Disminución en la ingesta de hierro

c) Exceso de hierro en médula ósea

d) Exceso de ferritina sérica

e) Mayor pérdida que asimilación de hierro

2. La anemia en este caso es:

a) Macrocítica e hipocrómica

b) Macrocítica e hipercrómica

c) Microcítica e hipocrómica

d) Microcítica y normocítica

e) Normocítica y normocrómica

3. El estudio más útil para establecer el tipo de anemia en esta paciente es:

a) Determinación de hemoglobina y hematócrito

b) Búsqueda de sangre oculta en heces fecales

c) Recuento de reticulocitos en sangre periférica

d) Proporción entre hierro sérico y fijación de hierro

e) Estudio morfológico de los eritrocitos

4. El soplo detectado en la paciente, se debe a:

a) Disminución del gasto cardiaco


b) Disminución de la hemoglobina

c) Disminución del hematócrito

d) Lesión orgánica valvular

e) Aumento del gasto cardiaco

5. El tratamiento indicado en este caso es:

a) Transfusiones sanguíneas

b) Sulfato ferroso por seis meses

c) Sulfato ferroso más hierro dextrán IV

d) Administración de hierro dextrán IV

e) Administración de hierro-dextrán IM

26 MEDICINA INTERNA
Masculino de 42 años de edad. Chofer de taxi, tabaquismo positivo a razòn de 20 cigarrillos diarios durante 24 años. Consume
alimentos en la calle. Inició su padecimiento un año antes con cuadros frecuentes de dolor retroesternal, opresivo, de moderada
intensidad, que se relacionan "cuando hace un coraje" o después de caminar muy rápido y cede con el reposo después de dos o
tres minutos. El motivo actual de consulta es la presencia de un episodio doloroso con las características descritas. Exploración
física: TA 180/100 mmHg, FC 80 por minuto, FR 18 por minuto. Consciente, plétora yugular grado I, campos pulmonares bien
ventilados, área cardíaca sin alteraciones. Abdomen globoso por abundante tejido adiposo. Extremidades sin alteraciones. ECG
de reposo normal.
1. Ante este cuadro clínico, el diagnóstico de presunción es:

a) Angina de Prinzmetal

b) Angina inestable

c) Angina estable

d) Angina clásica

e) Insuficiencia coronaria aguda

2. El factor etiológico que con mayor frecuencia se encuentra involucrado en estos casos es:

a) Hipertensión arterial

b) Tabaquismo

c) Arterioesclerosis

d) Obesidad

e) Sedentarismo

3. El auxiliar diagnóstico idóneo en este caso es:


a) Monitoreo Holter

b) Enzimas cardíacas

c) Ecocardiograma

d) Electrocardiograma

e) Prueba de esfuerzo

4. El cateterismo cardíaco probablemente mostrará:

a) Espasmo coronario

b) Obstrucción coronaria

c) Estenosis coronaria por ateromatosis

d) Dilatación de coronarias

e) Circulación colateral

5. El tratamiento médico inmediato incluye control de dolor y:

a) Heparina IV

b) Trombólisis

c) Nitroglicerina

d) Analgésicos no esteroides

e) Antiagregantes plaquetarios

27 MEDICINA INTERNA
Masculino de 22 años de edad, estudiante de cuarto año de la carrera de medicina. Presenta desde hace diez meses dolor
abdominal epigástrico, sordo y difuso, que a veces se calma con la ingesta de alimentos. Desde entonces ingiere ranitidina cuatro
o cinco días a la semana. Cuando la molestia se incrementa, utiliza gel de hidróxido de aluminio y magnesio. Hace seis meses se
agregaron náusea, vómito y dolores musculares generalizados. La exploración física muestra ginecomastia y dolor epigástrico a
la palpación del abdomen.
1. Con los datos anteriores, considera que uno de los problemas del paciente se debe a:

a) La interacción ranitidina antiácido

b) El estrés por sus estudios

c) Las reacciones adversas a la ranitidina

d) Las reacciones secundarias por el gel antiácido

e) La colonización por Helicobacter pylori


2. En tanto se realiza una serie gastroduodenal con prueba de aliento o una endoscopia, decide:

a) Cambiar a otro fármaco inhibidor de los H2

b) Ajustar la dosis de ranitidina

c) Indicar antiácidos

d) Prescribir un antiemético y un analgésico

e) Agregar un procinético

3. Con el diagnóstico radiológico de úlcera péptica y el informe positivo para H. pylori, usted indica el siguiente esquema
farmacológico-terapéutico, que ofrece 90% o más de alivio:

a) Omeprazol, subsalicilato de bismuto y claritromicina

b) Tetraciclina, metronidazol y bismuto

c) Claritromicina, metronidazol y omeprazol

d) Amoxicilina, claritromicina, bismuto y omeprazol

e) Amoxicilina, omeprazol y subsalicilato de bismuto

4. ¿Durante cuánto tiempo se recomienda el régimen terapéutico anterior? (en días)

a) 7

b) 10

c) 14

d) 21

e) 30

5. Las metas que pretende lograr con este tratamiento son las siguientes, EXCEPTO:

a) Evitar la recurrencia del cuadro

b) Favorecer la cicatrización de la úlcera

c) Erradicar H. pylori

d) Modificar los hábitos de vida

e) Prevenir las complicaciones de la enfermedad

28 MEDICINA INTERNA
Masculino de 44 años de edad. Procedente de su domicilio. Solicita atención de urgencia por dificultad respiratoria. Alcoholismo
crónico importante desde hace seis años llegando a la embriaguez cada semana. Pérdida de la fuerza muscular en las cuatro
extremidades desde hace seis meses. Inicia padecimiento actual hace tres días con fiebre de 39 a 40°C que se acompaña de tos.
Al examen físico se encuentra débil y con disnea de reposo, se auscultan estertores crepitantes en base pulmonar izquierda.
Radiografía de tórax con imagen de condensación basal izquierda. Leucocitosis de 14 100 por mm3 con neutrofilia.
1. Con base en el cuadro clínico usted establece el diagnóstico de:

a) Neumonía viral

b) Neumonía nosocomial

c) Neumonía comunitaria

d) Absceso pulmonar

e) Neumonitis alérgica exacerbada

Se dio tratamiento con penicilina G sódica y antitérmicos. Tres días después sigue febril, se deteriora el estado general y aparece
cianosis. La telerradiografía de tórax muestra imagen de derrame pleural izquierdo que se sobrepone a la condensación.
2. Con estos datos se sostiene el diagnóstico de:

a) Neumonía con SIRA

b) Neumonía con derrame pleural

c) Neumonía con derrame y empiema

d) Absceso pulmonar drenado

e) Empiema con derrame pleural

El estudio de líquido pleural informa presencia de piocitos y polimorfonucleares, pH; 5, glucosa de 25mg/dL, DHL de 2 000 UI.
3. Con estos nuevos datos debe pensar que:

a) El derrame pleural muestra complicación

b) El derrame pleural es propio de una neumonía

c) El derrame aparece por la contigüidad

d) El líquido es producto de un absceso

e) Se agregó pleuritis

La tomografía de tórax muestra derrame, consolidación neumónica y loculaciones.


4. El germen que frecuentemente ocasiona este padecimiento y la complicación agregada es:

a) Haemophilus influenzae

b) Streptococcus pneumoniae

c) Pseudomonas aeruginosa

d) Proteus vulgaris

e) Staphylococcus aureus
.
5. Para erradicar la infección y favorecer la reexpansión pulmonar lo recomendable en estos casos es:

a) Drenaje guiado por imágenes

b) Drenaje con sello de agua

c) Decorticación pleural

d) Toracocentesis frecuentes

e) Solamente medidas generales

29 MEDICINA INTERNA
Masculino de 30 años de edad. Antecedente de glomerulonefritis crónica. Padecimiento de cuatro meses de evolución
caracterizado por edema de miembros inferiores, no doloroso, simétrico, ascendente, de predominio matutino y edema
bipalpebral progresivo. Se acompaña de disnea de medianos a pequeños esfuerzos, nicturia, poliuria, astenia y adinamia. Desde
hace dos semanas se agrega tos con esputo blanquecino e irritabilidad. A la exploración física: TA 160/100 mmHg, FC 110 por
minuto, FR 26 por minuto, temperatura 36.7ºC, paciente en anasarca, confuso, con palidez generalizada, taquipneico,
taquicárdico, con frote pericárdico, estertores crepitantes bilaterales y ascitis libre. Laboratorio: Hb 7 g/dL, Hto 24%, leucocitos
4 300 por mm3, urea 224 mg/dL, creatinina 14 mg/dL, calcio 6.8 mg/dL.
1. En este paciente la poliuria y la nicturia se deben a:

a) Filtración glomerular disminuida

b) Concentración urinaria disminuida

c) Osmorreceptores sobreestimulados

d) Densidad urinaria aumentada

e) Compensación hemostática ácido/básica

2. La hipocalcemia es secundaria a:

a) Falta de acción de la vitamina D

b) Absorción de calcio disminuida

c) Excreción renal acelerada

d) Hipoalbuminemia

e) Falta de reabsorción tubular renal

3. El factor más importante en la patogenia de la hipertensión arterial es:

a) Ateroesclerosis

b) Expansión del volumen extracelular


c) Hipernatremia

d) Idiopático

e) Angiosclerosis renal

4. El tratamiento sustitutivo en la insuficiencia renal crónica, se efectúa generalmente cuando el filtrado glomerular es (en
porcentaje) de:

a) 20 - 25

b) 15 - 19

c) 10 - 14

d) 5 - 9

e) < 5

5. El tratamiento definitivo en este paciente es:

a) Diálisis peritoneal intermitente

b) Diálisis peritoneal continua ambulatoria

c) Hemofiltración intravenosa

d) Trasplante renal

e) Hemodiálisis

30 MEDICINA INTERNA
Femenina de 24 años de edad. Açude por incremento de 15 kg de peso durante los últimos cinco años. Su peso habitual era de 60
kg. En ocasiones sus periodos menstruales son irregulares. Niega otra sintomatología. Ingiere gran cantidad de grasa de origen
animal y carbohidratos. No realiza ejercicio. A la EF: TA 140/80 mmHg, peso: 74 kg, talla: 1.50 m. Obesa, adecuada coloración
de tegumentos, tiroides de tamaño normal, extremidades con datos de insuficiencia venosa leve.
1. El cuadro clínico anterior corresponde a un caso de:

a) Obesidad endógena

b) Obesidad exógena

c) Síndrome de ovarios poliquísticos

d) Síndrome de Cushing

e) Hipotiroidismo primario

2. En la práctica clínica el diagnóstico de obesidad se realiza mediante parámetros antropométricos, tomando en cuenta para el
IMC:

a) Talla y circunferencia de cintura


b) Talla y peso

c) Peso e índice cintura-cadera

d) Peso exclusivamente

e) Edad y peso

3. De acuerdo con la OMS se debe considerar con problema de obesidad al individuo cuyo índice de masa corporal (IMC)
calculado es:

a) Igual o mayor de 30

b) 25

c) 20

d) 15

e) Hasta 10

4. La etiología de esta entidad es:

a) Multifactorial

b) Ambiental

c) Transgresión dietética

d) Asociada a otra enfermedad

e) Genética predominante

5. El tratamiento inicial en este caso está encaminado a:

a) Colocación de anillo gástrico

b) Inhibidores de la recaptura de serotonina

c) Apoyo psicológico

d) Dieta, ejercicio y tratamiento farmacológico opcional

e) Administración de hormonas tiroideas

31 MEDICINA INTERNA
Acude a consulta masculino de 65 años de edad, tabiquero, fumador desde hace 40 años de 20 cigarrillos al día. Refiere varios
meses con tos productiva, expectoración hialina, sibilancias, disnea de pequeños esfuerzos, edema de miembros inferiores. Esta
sintomatologìa ha aumentado desde hace 15 días. A la exploración física: TA 140/80 mmHg, FC 75 por minuto, FR 24 por
minuto, afebril,rubicundez facial, obeso, con ingurgitación yugular, sin cianosis, murmullo vesicular disminuido, sibilancias
diseminadas, hipoventilación basal derecha, segundo ruido cardìaco intenso, soplo en tercero y cuarto espacios intercostales
izquierdos. Edema ++ y coloración ocre en miembros inferiores.
1. Con los datos anteriores, el diagnóstico más probable es:
a) Asma bronquial agudizado

b) Estenosis aórtica calcificada

c) Enfisema pulmonar

d) Bronquitis crónica

e) Corazón pulmonar crónico

2. En la telerradiografía de tórax esperarìa encontrar:

a) Imagen de hipertensión venocapilar

b) Radiotransparencia

c) Cardiomegalia y prominencia de los hilios

d) Trama broncovascular aumentada

e) Corazón en gota y abatimiento de hemidiafragmas

3. El electrocardiograma debe mostrar:

a) Bloqueo de rama izquierda

b) Fibrilación auricular lenta

c) Desnivel negativo del ST en DI, aVL, V5, V6 con T negativa

d) P acuminada en DII, DIII, aVF, patrón Rr en V1-2

e) P mitral y crecimiento ventricular derecho

4. El estudio gasomètrico informarà:

a) Hipoxemia e hipocapnia

b) Hipercapnia e hipoxemia

c) Hiperoxemia

d) Acidosis respiratoria

e) Alcalosis respiratoria

5. El tratamiento ideal para este paciente incluye:

a) Dopamina o dobutamina y diuréticos

b) Nitratos, diuréticos y broncodilatadores


c) lnhibidores de la ECA, diuréticos y digital

d) Antibióticos y oxigenoterapia

e) Antibióticos, broncodilatadores y oxígeno

32 MEDICINA INTERNA
Femenina de 40 años de edad. Con diagnóstico de SIDA, en tratamiento con antiretrovirales. Ingresa al hospital con pérdida
ponderal de 5 kg, fiebre 38ºC, deshidratación y diarrea de 20 días de evolución. A la exploración física, la paciente se encuentra
regularmente hidratada, TA de 90/60 mmHg, FC 90 por minuto.
1. La explicación fisiopatológica del síndrome diarreico de la paciente es:

a) Presencia de toxinas bacterianas

b) Inhibición de la reabsorción pasiva de Na+ y Cl- por bacterias

c) Inhibición de la reabsorción pasiva de Na+ y Cl- por inflamación

d) Aumento de la motilidad intestinal

e) Aumento en la secreción e inhibición de la absorción de Na+ y Cl-

2. Además de suero oral, el fármaco indicado para el manejo sintomático de este cuadro es:

a) Sulfasalazina

b) Caolín-pectina

c) Loperamida

d) Atropina

e) Prednisona

3. El fármaco seleccionado es útil por su acción como:

a) Adsorbente

b) Antiinflamatorio no esteroideo

c) Relajante de músculo liso intestinal

d) Agonista opioide

e) Antiinflamatorio esteroideo

4. Como efecto colateral del medicamento empleado, la paciente puede presentar:

a) Constipación y megacolon tóxico

b) Malestar gastrointestinal y sangre en heces


c) Sequedad bucal y cutánea

d) Disminución de la respuesta inmunológica de los linfocitos

e) Constipación y cólicos

5. La paciente está recibiendo zidovudina como tratamiento antiretroviral. Los principales efectos adversos que produce este
fármaco son:

a) Nerviosismo, dificultad para concentrarse y anorexia

b) Anemia y granulocitopenia

c) Hemólisis y supresión de la médula ósea

d) Náusea, diarrea e insuficiencia renal

e) Náusea y sabor amargo

33 PEDIATRIA
Lactante de ocho meses de edad, previamente sano, no ha iniciado esquema de inmunizaciones. Es llevado al hospital por
presentar fiebre y respiración rápida. A la exploración física, con temperatura de 38.8°C y FR 70 por minuto, Silverman 2-3 a
expensas de tiraje intercostal, submatidez media derecha, vibraciones vocales disminuidas y crépitos gruesos diseminados en
ambos campos pulmonares de predominio derecho. La radiografía P.A. de tórax reveló opacidad del lóbulo medio e inferior del
lado derecho, con mínimo derrame pleural. La biometría hemática reportó 25 000 leucocitos por mm3.
1. Para confirmar su diagnóstico ¿qué otro estudio solicita?

a) Cultivo de exudado faríngeo

b) Cultivo de esputo

c) Velocidad de sedimentación globular

d) Prueba de aglutinación en látex

e) Radiografía lateral de tórax

2. El agente causal más probable que participa en este padecimiento es:

a) Haemophilus influenzae tipo b

b) Klebsiella pneumoniae

c) Streptococcus pneumoniae

d) Estreptococo del grupo B

e) Escherichia coli

3. ¿Qué antimicrobiano está indicado para este paciente?

a) Penicilina sódica
b) Eritromicina

c) Ampicilina

d) Cefotaxima

e) Dicloxacilina

4. El tiempo mínimo indicado de terapia antimicrobiana para este caso es de (en días):

a) 5

b) 7

c) 14

d) 21

e) 28

5. Otra medida terapéutica a emplear en este caso es:

a) Corticoesteroides

b) Digital

c) Aminofilina

d) Punción pleural evacuadora

e) Oxígeno y terapia respiratoria integral

34 PEDIATRIA
Masculino de 10 meses de edad, que es llevado al servicio de urgencias por padecimiento actual de dos días de evolución,
caracterizado por fiebre de 39.5°C, tos productiva, emetizante, con expectoración verdosa. Exploración física: palidez
generalizada ++++, faringe hiperémica, cuello con adenomegalias de 0.5 a 1 cm., campos pulmonares sin alteraciones, soplo
sistólico plurifocal, abdomen normal. Cuenta con los siguientes antecedentes de importancia: alimentado con leche entera a
partir del cuarto mes de vida con siete biberones de 8 onzas en 24 horas. Ablactación a los 6 meses de edad a base de frutas,
verduras y cereales. Laboratorio: BH Hb 6g/dL, hematócrito 20%, leucocitos 8 200/mm3, segmentados 75%, linfocitos 20%,
monocitos 5%, plaquetas 850 000/mm3, reticulocitos 3%, VCM 55, CMH 23.
1. Con base en el cuadro clínico descrito, el diagnóstico más probable es:

a) Anemia megaloblástica

b) Anemia por deficiencia de hierro

c) Trombocitosis esencial

d) Aplasia pura de serie roja

e) Talasemia
2. Esta entidad se presenta con mayor frecuencia a la edad de:

a) 3-4 meses

b) 7-9 meses

c) 12-18 meses

d) 2 años

e) 5 años

3. El paciente es transfundido para incrementar la hemoglobina a 7.5 g. Se debe continuar tratamiento con:

a) Vitamina B12 1mcg/24 horas

b) Ácido fólico 5 mg cada 24 horas

c) Prednisona 2 mg/Kg./día

d) Hierro dextrán IM. 50 mg/dosis

e) Hierro elemental 5 mg/Kg./día

4. La duración del tratamiento es aproximadamente de:

a) 15 días

b) 1-2 meses

c) 4-6 meses

d) 8-10 meses

e) 12 meses

5. El primer parámetro de laboratorio en corregirse es:

a) El volumen corpuscular medio

b) La amplitud de distribución eritrocitaria

c) La cuenta de reticulocitos

d) La cifra de hemoglobina

e) La concentración media de hemoglobina

35 PEDIATRIA
Masculino de seis meses de edad, proviene de medio socioeconómico bajo. Peso al nacer de 2.800 Kg, talla 49 cm. Alimentado
con leche materna por dos meses, posteriormente con leche LICONSA, ablactado a los tres meses con papillas de frutas y
verduras hasta el momento. Exploración física: peso 5.800 Kg, talla 61 cm, pálido ++, irritable al manejo, buen estado de
hidratación. Resto de la exploración física normal. Exámenes de laboratorio: Hb 10 g/dL, Hto 30%, leucocitos 6 000 por mm3,
neutrófilos 55%, linfocitos 43%, eosinófilos 2%, reticulocitos 1%. Frotis con microcitos, hipocromía y anisocitosis, proteínas
séricas normales. Química y electrólitos séricos normales.
1. Con los datos antes mencionados, se determina que el niño se encuentra con:

a) Buen estado nutricio

b) Somatometría limítrofe

c) Desnutrición de primer grado

d) Desnutrición de segundo grado

e) Desnutrición de tercer grado

2. ¿Qué factor(es) determina(n) fisiológicamente el inicio de la ablactación?

a) Sociales, económicos y familiares

b) Inicio de la dentición

c) Tipo racial

d) Madurez renal, neurológica y gastrointestinal

e) Madurez pancreática y pulmonar

3. La deficiencia del hierro se manifiesta por anemia de tipo:

a) Macrocítica hipocrómica

b) Microcítica hipocrómica

c) Microcítica hipercrómica

d) Poiquilocítica

e) Megaloblástica

4. El paciente presenta anemia carencial por la deficiencia de hierro. Los factorespredisponentes en este caso son los siguientes
EXCEPTO:

a) Destete temprano

b) Uso de leche de vaca y escasas reservas de hierro

c) Somatometría limítrofe al nacer

d) Nivel sociocultural bajo

e) Inicio de ablactación con papillas de frutas y verduras

5. Para corregir la deficiencia de hierro, el tipo de alimento que recomienda para este paciente es leche:

a) De soya y estimular ablactación con vegetales


b) Semidescremada y cereales

c) Semidescremada y hierro suplementario

d) Entera adicionada con hierro y complementada con vegetales y carnes

e) Maternizada y hierro suplementario

36 PEDIATRIA
Femenina de un mes de edad. Es llevada a consulta por su madre para control del niño sano. Antecedentes de importancia;
nacida por cesárea debido a presentación pélvica. Exploración física: presenta bloqueo de la cadera derecha para la rotación
externa con signo de Ortolani positivo y Savariaud positivo.
1. De acuerdo con los datos anteriores, el diagnóstico más probable es:

a) Displasia acetabular

b) Insuficiencia femoral

c) Hipoplasia del núcleo de osificación del fémur

d) Cadera luxada de tipo congénito

e) Inestabilidad progresiva de la cabeza femoral

2. ¿Qué estudio(s) paraclínico(s) solicita para confirmar su diagnóstico?

a) Biometría hemática y química sanguínea

b) Niveles hormonales séricos

c) Artrografía de cadera

d) Radiografía simple de pelvis en posición neutra

e) Resonancia magnética nuclear de cadera

3. La conducta a seguir en esta paciente es:

a) Colocación de doble pañal y dejarla en observación

b) Colocación de doble pañal y se canaliza al servicio de pediatría

c) Colocación de una ortesis como cojín de Freckja o Arnés de Pavlick

d) La cita con nuevos estudios hasta el mes siguiente

e) La canaliza de urgencia al servicio de ortopedia

4. El signo de Galeazzi consiste en:

a) Movilidad anormal cuando la extremidad es impulsada con la cadera en abducción


b) Acortamiento del fémur juzgado por la diferencia en el nivel de las rodillas

c) Que cuando el niño ya camina, presenta "marcha de pato"

d) Que se doblan las rodillas, las caderas se flexionan a 90° y se colocan en abducción total

e) Que se observa radiológicamente un desplazamiento del núcleo femoral

5. La complicación por maniobras de reducción forzada o por tensión de tejidos blandos periarticular es:

a) Inversión de Lambrum

b) Ruptura del ligamento redondo

c) Necrosis avascular

d) Fractura de ceja acetabular

e) Fractura transcervical del fémur

37 PEDIATRIA
Femenina de cuatro años de edad. Hace dos semanas presentó rinofaringitis. La sintomatología cedió en cinco días con
tratamiento de ácido acetilsalicílico y difenhidramina. Esquema de inmunizaciones al corriente de acuerdo con su edad. Se
refiere que la niña es "muy juguetona" y ha presentado diversos traumatismos. Se tomó radiografía de la rodilla hace una
semana. El día de hoy presenta repentinamente petequias diseminadas en forma asimétrica, principalmente en tobillos y región
tibial anterior, donde se observan además dos equimosis. Cursa con cefalea y náusea. A la exploración física: temperatura
37.3°C, se corroboran las lesiones cutáneas descritas, hepatomegalia 2 cm por debajo del borde costal. Laboratorio: 7 200
leucocitos/mm3, eosinofilos 7%, 4.1 millones de eritrocitos/mm3, Hb 12.4 g/dL, 20 000 plaquetas/mm3.
1. Con base en el cuadro clínico descrito, el diagnóstico más probable es:

a) Púrpura por medicamentos

b) Púrpura de Henoch-Schönlein

c) Púrpura trombocitopénica idiopática

d) Leucemia linfoblástica aguda

e) Síndrome de Fanconi

2. El criterio que apoya su diagnóstico es:

a) Uso previo de antihistamínicos y ácido acetilsalicílico

b) Trombocitopenia aislada

c) Hepatomegalia

d) Distribución de las petequias

e) Presencia de cefalea

3. Para apoyar su diagnóstico solicita los siguientes exámenes paraclínicos, EXCEPTO:


a) Punción-biopsia de médula ósea

b) Vida media plaquetaria

c) Volumen plaquetario medio

d) Tiempo de sangrado y retracción del coágulo

e) Cuantificación de reticulocitos

Se repite en siete días la cuenta de plaquetas y es de 18 000/mm3


4. El tratamiento indicado en este caso es:

a) Gammaglobulina intravenosa

b) Prednisona

c) Esquema MOPP

d) Plasma fresco

e) Concentrado plaquetario

5. El pronóstico usual en esta entidad es:

a) La mortalidad es de 10 a 20%

b) 10% tendrá cuenta de plaquetas normal en cuatro semanas

c) 50% presentará la forma crónica

d) 70% presentará la forma recurrente

e) 90% se recupera en seis meses

38 URGENCIAS
Masculino de 35 años de edad. De oficio electricista. Recibió un flamazo al estar manipulando un transformador de corriente, lo
que ocasionó pérdida temporal del estado de alerta. A la exploración física: TA 110/50 mmHg, FC 120 por minuto, FR 30 por
minuto y 38.5°C de temperatura. Consciente, orientado y con facies de dolor. Presenta quemaduras de segundo grado, profundas,
en cabeza, cara, cuello, cara anterior de tórax, brazos y abdomen. Los ruidos cardíacos se encuentran rítmicos y se ausculta
rudeza respiratoria. El resto de la exploración sin alteraciones.
1. La conducta inicial en este paciente debe ser:

a) Canalizar vías venosas central y periférica

b) Tomar muestras sanguíneas para laboratorio

c) Lavado de las lesiones

d) Asegurar vía aérea permeable


e) Solicitar radiografía de tórax

2. Para el tratamiento de este paciente, debe tomar en cuenta principalmente:

a) La profundidad de las lesiones

b) La extensión de las lesiones

c) La causa que ocasionó las lesiones

d) La edad del paciente

e) El tiempo de evolución

3. Las soluciones utilizadas para la restitución de volumen de este paciente,deben contener concentración suficiente de:

a) Glucosa

b) Bicarbonato

c) Sodio

d) Potasio

e) Cloro

4. Una vez estabilizado el paciente, la conducta prioritaria a seguir es:

a) Apoyo nutricional

b) Cirugía reconstructiva

c) Corticoesteroides a dosis mínimas

d) Administrar inmunoglobulinas

e) Mantener la piel no dañada en buenas condiciones para tomar injertos

5. La causa más frecuente de muerte, a mediano plazo, en este tipo de pacientes es:

a) Desequilibrio hidroelectrolítico

b) Infección

c) Desnutrición

d) Insuficiencia renal aguda

e) Pancreatitis aguda

39 URGENCIAS
Masculino de 30 años de edad. Impactó su auto contra un poste al conducir con exceso de velocidad. Se golpeó el tórax y
abdomen contra el volante y la cabeza contra el parabrisas. A la exploración física, se encuentra en estado de confusión mental,
con aliento alcohólico, pálido, diaforético, TA 60/40 mmHg, FC 140 min, FR 32 min y temperatura 35.5°C; los ruidos
respiratorios se encuentran velados en hemitórax derecho y el resto de la exploración sin datos de interés.
1. En este paciente la causa más probable de la confusión mental es:

a) Intoxicación etílica

b) Traumatismo craneoencefálico

c) Hipoventilación pulmonar

d) Tamponade

e) Choque hipovolémico

Dentro de las medidas terapéuticas, decide colocar una sonda de Foley al paciente.
2. ¿Cuál es el flujo urinario (mL/kg/hora) esperado en un paciente adulto normal?

a) 0.3

b) 0.5

c) 1.0

d) 2.0

c) 3.0

3. La alteración celular ocasionada por la hipoperfusión tisular en el paciente es:

a) Energetización de las mitocondrias

b) Cambio de metabolismo aeróbico por anaeróbico

c) Aumento de liberación de energía

d) Aumento del metabolismo aeróbico

e) Pérdida de agua

4. La solución que debe administrarse a este paciente para iniciar el restablecimiento de la perfusión es:

a) Glucosada a 5%

b) Soluciòn fisiològica

c) Ringer lactado

d) Plasma

e) Mixta con vasopresores

5. De todos los signos que integran el estado de choque, el primero que se presenta es:
a) Hipotensión

b) Taquicardia

c) Diaforesis

d) Cianosis

e) Confusión mental

40 URGENCIAS
Masculino de 62 años de edad. Antecedentes de diabetes mellitus e hipertensión arterial de 30 años de evolución, controlado con
glibenclamida, captopril y furosemida; posoperado de aneurisma aórtico hace cinco años, por lo que toma antiagregante
plaquetario. Hace tres meses se agregó alopurinol por presentar cifras elevadas de ácido úrico. Refiere pérdida de peso de cinco
kilogramos en los últimos doce meses. Inicia padecimiento actual con evacuaciones melènicas en número de ocho durante las
últimas 12 horas, además de náusea y astenia. Niega otra sintomatología. A la exploración física: TA 100/55 mmHg, FC 105 por
minuto, FR 25 por minuto, temperatura 36.0ºC. Se encuentra pálido y diaforético. El abdomen es plano, depresible y doloroso a
la palpación media y profunda de epigastrio, no hay rebote y la peristalsis se encuentra aumentada.
1. Para iniciar la administración de líquidos en este paciente se debe colocar:

a) Catéter subclavio

b) Catéter largo

c) Punzocat No. 18

d) Punzocat No. 14

e) No existe diferencia

2. Se coloca sonda nasogástrica y se corrobora la hemorragia. Èsta provoca la activación de la:

a) Gastrina

b) Pepsina

c) Histamina

d) Acetilcolina

e) Adrenalina

3. El límite anatómico para clasificar la hemorragia de este paciente es:

a) La válvula ileocecal

b) La tercera porción del duodeno

c) El esfínter pilórico

d) El ligamento de Treitz
e) El yeyuno

4. La causa más frecuente de hemorragia de tubo digestivo alto es:

a) Úlcera gástrica

b) Úlcera duodenal

c) Hemobilia

d) Várices esofágicas

e) Esofagitis erosiva

5. Se realiza panendoscopia, la cual informa gastritis erosiva hemorrágica. Este tipo delesiones son poco frecuentes en:

a) Curvatura mayor

b) Curvatura menor

c) Fondo

d) Cuerpo

e) Antro

41 URGENCIAS
Femenina de 19 años de edad. Refiere dolor intenso en el cuadrante inferior derecho del abdomen, náusea y vómito. Su última
menstruación fue hace seis semanas con duraciòn de cuatro dìas y caracterìsticas habituales. A la exploración física se encuentra
con TA 90/50 mmHg FC 120 por min FR 25 por min, temp 35 C. Llenado capilar de 4 seg, somnolienta y diaforètica. Exámenes
con prueba de embarazo positiva y ultrasonido pélvico que muestra masa anexial derecha, abundante líquido libre en cavidad y
útero vacío. Inicialmente se administran soluciones parenterales y posteriormente se decide transfundir un paquete globular. La
paciente manifiesta dolor en el brazo canalizado, región lumbar y tórax. Presenta escalofrío, fiebre y oliguria. Continua con
hipotensión.
1. Con los datos anteriores, debe sospechar una reacción:

a) A sangre contaminada

b) A pirógenos

c) Hemolítica

d) Febril alérgica

e) Anafiláctica

2. La medida inmediata que debe tomar es:

a) Incrementar aporte de sangre

b) Detener la transfusión

c) Administrar antihistamínicos
d) Administrar corticoesteroides

e) Cirugía de inmediato

3. Para confirmar el diagnóstico de la complicación, necesita:

a) Determinar si hay hemoglobinuria

b) Pruebas de función hepática

c) Gases arteriales

d) Fórmula roja seriada

e) Hemocultivo

4. La complicación de la paciente fue ocasionada por la siguiente causa:

a) Atraso en la toma de decisión

b) Sangre almacenada pobre en elementos de coagulación

c) Contaminación del equipo

d) Contaminación de la sangre

e) Incompatibilidad del grupo sanguìneo

20. Esta paciente tiene altas probabilidades de presentar daño tubular renal. Con el objeto deevitar esta complicación, es
necesario administrar:

a) Manitol y bicarbonato

b) Dextrán

c) Solución glucosada

d) Solución de Hartman

e) Acidificantes urinarios

42 URGENCIAS
Masculino de 23 años de edad. Es llevado a urgencias por sus familiares, tras haber ingerido una cantidad desconocida de
tabletas, al parecer analgésicos. A la exploración física: TA 110/60 mmHg, FC 80 por minuto, FR 16 por minuto, eutérmico; con
Glasgow de 9 (2-3-4), pupilas puntiformes, regularmente hidratado. Respiración lenta y superficial, con cianosis distal y
peribucal, pulso débil y espasmo muscular generalizado. Presenta crisis tonicoclónica en el servicio. Se administra diazepam 10
mg con lo que cede la crisis.
1. Los datos clínicos del paciente corresponden a una intoxicación por:

a) Acido acetilsalicílico
b) Paracetamol

c) Propoxifeno

d) Metocarbamol

e) Piroxicam

2. Para confirmar la sospecha diagnóstica es indispensable realizar:

a) Pruebas de función hepática

b) Biometría hemática

c) Química sanguínea

d) Pruebas cualitativas

e) Pruebas cromatográficas

3. La combinación del medicamento causante de la intoxicación con inhibidores de la monoaminooxidasa, puede causar:

a) Coma con normotermia y edema podálico

b) Coma con hipotermia y edema angioneurótico

c) Coma con hipertermia y edema cerebral

d) Hipotermia general y edema hepático

e) Hipotermia y edema generalizado

4. En el manejo en urgencias es prioritario:

a) Lavado gástrico a través de sonda de Levin

b) Oxígeno 3 litros por minuto por puntas nasales

c) Colocar sonda Foley y cuantificar uresis

d) Realizar exámenes de laboratorio y gabinete

e) Mantener la vía aérea permeable

5. El antídoto específico en este caso es:

a) Levoproxifeno

b) Naloxona

c) Levalorfan
d) Flumacenid

e) N Acetilcisteína

43 URGENCIAS
Masculino de 52 años de edad. Antecedente de tabaquismo intenso desde los 15 años a razón de 24 cigarrillos al dìa.
Hipertensión arterial sistémica desde hace 10 años, diabetes mellitus desde hace seis. Inicia padecimiento hace seis meses con
dolor opresivo en cara anterior de tórax, intenso, en reposo, irradiado a cuello y brazo izquierdo, con duración inicial de tres
minutos. Aumenta en intensidad y frecuencia de manera progresiva. El día de su internamiento el dolor duró cinco horas,
acompañado de palidez, diaforesis intensa, cansancio, disnea y palpitaciones. A la exploración física: con FC de 90 por minuto,
TA 90/60 mmHg pálido, diaforético. Tórax en tonel, con hipersonoridad pulmonar, con escasos estertores crepitantes basales.
Con tercer ruido en área precordial. Laboratorio: enzimas séricas; TGO 55 U/L, CPK 400 U/L, fracción MB 120 U/L. Hb 18
g/dL,Hto 54%, leucocitos 12 700 por mm3 con 80% de segmentados, 20% linfocitos. Glucemia 180mg/dL, urea 32 mg/dL,
creatinina 1.0 mg/dL, colesterol 400 mg/dL, triglicéridos 300 mg/dL,ácido úrico 10.0 mg/dL. ECG: ritmo de base sinusal,
extrasístoles ventriculares aisladas; frecuencia ventricular promedio 90 por minuto. Bloqueo fascicular anterior
izquierdo,BARDHH, onda "Q" de V1 a V4 con supradesnivel del segmento ST-T. Hipertrofia ventricularizquierda. Radiografía
de tórax: cardiomegalia grado II, sin signos de hipertensión venocapilarpulmonar, aortosclerosis. Signos de enfisema pulmonar.
1. La manifestación inicial de la cardiopatía isquémica en este caso fue:

a) Infarto agudo del miocardio

b) Isquemia miocárdica silenciosa

c) Angina estable

d) Angina inestable

e) Disnea, acompañada de palpitaciones y cansancio

2. La etiología del infarto del miocardio es:

a) Exclusivamente aterosclerosis coronaria

b) Embolia coronaria

c) Aterosclerosis coronaria con trombosis de la misma

d) Aterosclerosis coronaria con embolia

e) Aumento de agregación plaquetaria in situ

3. El factor de riesgo más importante en este paciente es:

a) Edad

b) Sexo

c) Hipertensión arterial sistémica

d) Tabaquismo

e) Dislipidemia

4. Los signos electrocardiográficos que corresponden a infarto agudo del miocardio en esteenfermo son:
a) BARDHH más BFAI más extrasístoles ventriculares

b) BARDHH más BFAI e infradesnivel ST en cara diafragmática

c) Presencia de onda Q con bloqueo avanzado de RDHH más infradesnivel del segmento ST-T de V1 a V4

d) Bloqueo avanzado de la rama derecha del Haz de His y presencia de onda Q

e) Extrasístoles ventriculares más BFAI y supradesnivel ST en cara anterior

5. Por las alteraciones electrocardiográficas referidas la localización del infarto es:

a) Cara posterior

b) Cara anterior

c) Cara anterior extensa

d) Anteroseptal

e) Pared libre del ventrículo izquierdo

44 URGENCIAS
Masculino de 47 años de edad. Acude a urgencias por disnea progresiva hasta ortopnea en 4 días, tos con expectoración
blanquecina abundante, palpitaciones y edema de miembros pélvicos hasta su base. Antecedentes de tabaquismo intenso,
hipertensión arterial y diabetes mellitus de larga evolución y control irregular. A la EF: TA 130/70 mmHg, FC 110 por minuto,
cianosis distal +, plétora yugular G1, estertores crepitantes diseminados en campos pulmonares, área precordial con soplo
sistólico en foco mitral G II/VI, hepatomegalia, edema ++ hasta el muslo. ECG imágenes de QS de V1 a V4, T invertida
simétrica, R alta en V5, V6, D1 a VL. Laboratorio: glucosa 130 mg/dL, creatinina 1.1 mg/dL, urea 20 mg/dL, Hb 17.0 g/dLm,
Hto 50%, Rx tórax: índice cardiotorácico de 52%, hilios empastados y redistribución de flujo.
1. De los siguientes diagnósticos el de mayor importancia a su ingreso es:

a) Edema agudo pulmonar

b) Insuficiencia cardíaca congestiva venosa

c) Insuficiencia mitral grave

d) Tromboembolia pulmonar

e) Infarto agudo del miocardio

2. La causa desencadenante principal del cuadro clínico en este caso es:

a) Hipertensión arterial

b) Diabetes mellitus descontrolada

c) Dislipidemia

d) Cardiopatía isquémica
e) Cardiomiopatía dilatada primaria

3. El estudio NO invasivo que permite evaluar de manera más adecuada el estado funcional del paciente es:

a) Electrocardiograma

b) Serie cardíaca

c) Holter de electrocardiografía

d) Pruebas de función respiratoria

e) Ecocardiograma

4. El estudio indicado para establecer con mayor precisión un diagnóstico y pronóstico en este caso es:

a) Prueba de esfuerzo en banda

b) Prueba de inclinación

c) Prueba de perfusión miocárdica

d) Diferencia arteriovenosa de oxígeno

e) Gasto cardíaco con catéter de flotación

5. El tratamiento inicial para este paciente es:

a) Digital, diurético, antiagregante plaquetario e inhibidor de la ECA

b) Digital, beta-bloqueador, diurético y antiagregante plaquetario

c) Beta-bloqueador, calcio-antagonista, antiagregante plaquetario y diurético

d) Diurético, beta-bloqueador, digital y calcio antagonista

e) Anticoagulación con heparina IV

45 URGENCIAS
Femenina de 23 años de edad. Cursa puerperio fisiológico mediato. Cuadro clínico de 12 horas de evolución que inició con dolor
súbito en hemitórax derecho, de mediana intensidad, acompañado de disnea de mínimos esfuerzos, tos seca, aislada. Exploración
física: TA 120/80 mmHg, FC 106 por minuto, FR 30 por minuto, temperatura 36°C. Se observa disneica, angustiada, pálida. En
la exploración del tórax no se encuentran datos anormales; la radiografía del tórax muestra elevación del hemidiafragma
derecho; electrocardiograma con taquicardia sinusal. Gasometría con datos de hipoxemia e hipocapnia. Las fórmulas roja y
blanca sin alteraciones.
1. Con los datos obtenidos el diagnóstico más probable es:

a) Sìndrome de Meigs

b) Tromboembolia pulmonar

c) Neumotórax espontáneo
d) Neumonìa bacteriana

e) Neumonìa viral

2. El diagnóstico de insuficiencia respiratoria se basa en:

a) La presencia de disnea

b) La presencia de cianosis ungueal y labial

c) La utilización de los músculos accesorios de la respiración

d) La respuesta positiva a una prueba terapéutica

e) Los resultados de la gasometría arterial

3. La hipoxemia que se observa en este caso es causada por:

a) Hipercapnia

b) Hipocapnia

c) Broncoconstricción

d) Pérdida de surfactante

e) Alteración en la relación ventilación/perfusión

4. El estándar de oro para el diagnóstico del padecimiento de base, en este caso, es:

a) La radiografía del tórax

b) La gasometría arterial

c) Los niveles séricos de DHL

d) La angiografía pulmonar

e) La gammagrafía pulmonar

5. El tratamiento de la insuficiencia respiratoria crónica en pacientes ambulatorios es:

a) Oxígeno por puntas nasales

b) Oxígeno con fracción inspirada de 40%

c) Broncodilatadores

d) Inhaloterapia

e) Expectorantes y mucolíticos
46 CIRUGIA
Femenina de 34 años de edad. Refiere dolor en cuello desde hace tres meses con dificultad para la deglución, aumento de
volumen de cuello y tumoración en la parte lateral derecha, así como pérdida de peso, insomnio e intolerancia al calor. Paciente
consciente, pulso 110 por minuto, cuello con tiroides aumentada de volumen en lóbulo derecho, móvil a la deglución, no se
palpa crecimiento de ganglios cervicales.
1. Con los datos clínicos mencionados ¿cuál es el diagnóstico más probable?

a) Bocio simple

b) Hipertiroidismo

c) Hipotiroidismo

d) Cáncer tiroideo

e) Tiroiditis de Riedel

2. Para confirmar su diagnóstico, el estudio más útil es:

a) Gammagrama

b) T4

c) T3 inversa

d) T3 libre

e) Tiroglobulina

3. En caso de encontrar pruebas de función tiroidea con T4 de 13 microg/dL, T3 de 250 microg/dL y TSH de 40 microg/dL ¿en
qué piensa?

a) Eutiroidismo

b) Hipotiroidismo

c) Hipertiroidismo

d) Tiroiditis

e) Cáncer tiroideo

4. ¿Cuál es el tratamiento inicial en este caso?

a) Tiroidectomía radical

b) L- Tiroxina, 100 microgramos c/24 horas

c) Propiltiouracilo y propranolol

d) Tiroidectomía subtotal

e) Yodo radiactivo, 50 microgramos dosis única


5. En caso de que el tratamiento funcione pobremente ¿cuál sería ahora su manejo?

a) Hemitiroidectomía

b) Propiltiouracilo y metimazol

c) Yodo radiactivo o tiroidectomía

d) Propiltiouracilo y propranolol

e) Solamente propranolol

47 CIRUGIA
Femenina de 71 años de edad. Tabaquismo positivo desde los 20 años a razón de veinte cigarrillos al día. Diabética desde hace
14 años en control con hipoglucemiantes orales. Hipertensa desde hace 12 años en control con amlodipino y metoprolol.
Enfermedad osteoarticular por la que ingiere en forma crónica diclofenaco e ibuprofeno. Toma digital y ácido acetilsalicílico
desde hace cuatro años, debido a infarto agudo del miocardio. Inició cinco días antes de su ingreso al hospital con dolor urente
en epigastrio, que la despierta por la madrugada, sin irradiación, sensación de hambre dolorosa que disminuye con la ingesta de
alimento, para reaparecer una a dos horas más tarde, acompañada de náusea sin llegar al vómito; palpitaciones, mareo y
desvanecimiento sin pérdida del estado de alerta. A la exploración física TA 90/70 mmHg, FC 98 por minuto. Consciente, con
palidez de tegumentos y diaforesis; regularmente hidratada. Abdomen con dolor a la palpación profunda de epigastrio, sin datos
de irritación peritoneal. Tacto rectal con datos de melena.
1. La conducta inicial en este caso es:

a) Instalar sonda de balón

b) Instalar sonda nasogástrica

c) Intubación orotraqueal

d) Aplicar soluciones cristaloides

e) Tomar electrocardiograma

La biometría hemática informa Hb 7.8 g/dL, Hto 22.3%, con índices eritrocitarios normales.
2. Lo indicado en esta situación es:

a) Lavado gástrico con solución fisiológica

b) Instalar sonda Foley

c) Transfundir paquetes globulares

d) Polimerizado de gelatina

e) Soluciones coloides

3. El diagnóstico endoscópico fue úlcera duodenal activa con hemorragia rezumante. El procedimiento terapéutico que debe
realizarse es:

a) Ninguno específico

b) Ligadura de vaso visible


c) Escleroterapia

d) Toma de biopsia perilesional

e) Dilatación con balón

4. El tratamiento indicado en este caso es:

a) Dieta antipéptica

b) Antisecretores

c) Antiácidos

d) Procinéticos

e) Anticolinérgicos

5. La cirugía de urgencia está indicada cuando:

a) La úlcera duodenal está activa

b) La úlcera duodenal es mayor de 2 cm de diámetro

c) La úlcera duodenal cursa con hemorragia activa que remitió con escleroterapia

d) La úlcera duodenal se encuentra cicatrizada

e) La paciente requiere más de cuatro paquetes globulares en menos de 24 horas

48 CIRUGIA
Masculino de 23 años de edad. Antecedente de alcoholismo ocasional. Acude al servicio de urgencias por presentar un cuadro de
18 horas de evolución, caracterizado por dolor moderado a intenso, localizado en mesogastrio, flanco y fosa iliaca derechas, con
una evacuación semilíquida, temperatura de 37.8°c, náusea y vómito de contenido gastroalimentario. Hiporexia. A la exploración
física se encuentra consciente, orientado, bien hidratado. Abdomen con hiperestesia e hiperbaralgesia en fosa iliaca derecha.
Peristalsis aumentada +.
1. Un factor predisponente para la presentación de esta patología es:

a) Insuficiencia vascular

b) Linfadenitis

c) Obstrucción de la luz

d) Sepsis

e) Necrosis

2. La causa más frecuente de esto, es:

a) Taponamiento por bario


b) Hipertrofia linfoide

c) Cuerpos extraños

d) Fecalitos

e) Parásitos

3. El diagnóstico del padecimiento se fundamenta en:

a) Hallazgos de imagenología y laboratorio

b) Biometría hemática y pruebas de coagulación

c) Historia clínica

d) Ultrasonido abdominal

e) Colon por enema

4. En niños, debe hacerse diagnóstico diferencial con:

a) Pielonefritis

b) Divertículo de Meckel

c) Adenitis mesentérica

d) Cistitis

e) Parasitosis

5. El uso de antibióticos en forma profiláctica se indica:

a) Durante la cirugía

b) Antes, durante y posterior a la cirugía

c) Desde que se determina el tratamiento quirúrgico

d) Durante 10 días

e) Únicamente en la fase complicada

49 CIRUGIA
Masculino de 57 años de edad. Antecedente de hipertensión arterial de 12 años de evolución, controlado con inhibidores de la
ECA. Acude con dolor de tipo punzante, constante, localizado en la región inguinoescrotal derecha, sin irradiaciones, con
sensación de plenitud y distensión abdominal, de 12 horas de evolución. Asimismo refiere la presencia de tumoración en región
inguinal derecha desde hace tres meses, que aparece generalmente con esfuerzos y que reduce manualmente, diagnosticada como
hernia inguinal y tratada conservadoramente con faja. En esta ocasión no es posible la reducción, por lo que acude al hospital. A
la exploración física se encuentra con palidez de tegumentos y regular estado de hidratación, con actitud antiálgica. FC 95 por
minuto, TA 140/110 mmHg, FR 20 por minuto, temperatura 38.0°C. Con dolor a la palpación de hemiabdomen inferior, con
aumento de volumen en región inguinoescrotal derecha, de coloración violácea, consistencia dura, edematosa y muy dolorosa, no
es reductible y presenta crepitación. La peristalsis se encuentra disminuida.
1. Con los datos clínicos, la clasificación correcta de esta hernia es:

a) Directa incarcerada

b) Directa estrangulada

c) Indirecta incarcerada

d) Indirecta estrangulada

e) Por deslizamiento estrangulada

2. Para realizar el diagnóstico anterior, se debe solicitar:

a) Herniografía

b) Placas simples de abdomen

c) El examen físico es suficiente

d) Ultrasonograma inguinal

e) Transiluminación

3. A la zona limitada por la arteria epigástrica inferior, el ligamento inguinal y el borde lateral de la vaina del recto, se le
denomina:

a) Anillo inguinal profundo

b) Anillo inguinoescrotal

c) Anillo inguinal superficial

d) Triángulo de Hesselbach

e) Triángulo femoral de Scarpa

4. A la hernia que sobresale a través del piso del canal inguinal, en el triángulo de Hesselbach, se denomina:

a) De Richter

b) De Littré

c) Por deslizamiento

d) Indirecta

e) Directa

5. Para realizar el diagnóstico de estrangulación en este paciente, debe existir:

a) Edema local
b) Irritación peritoneal

c) Oclusión intestinal

d) Compromiso vascular

e) Irreductibilidad

50 CIRUGIA
Femenina de 60 años de edad. Antecedente de diabetes mellitus de 10 años de evolución, con tratamiento médico en forma
irregular y el cual suspendió por completo hace un mes. Refiere iniciar su padecimiento actual hace aproximadamente 24 horas
con dolor tipo cólico localizado en mesogastrio, el cual fue generalizándose y aumentando de intensidad, se agregaron náusea y
vómito en varias ocasiones, de contenido gastrobiliar inicialmente y posteriormente fecaloide, además de fiebre no cuantificada.
Niega evacuaciones y canalización de gases. A la exploración física: FC 120 por minuto, FR 28 por minuto, TA 100/55 mm/Hg,
temperatura 38.2°C. Se encuentra somnolienta, con piel y mucosas deshidratadas. El abdomen se encuentra distendido, con
hiperestesia, hiperbaralgesia y dolor generalizado a la palpación, hay timpanismo y la peristalsis se encuentra ausente. Al tacto
rectal con ámpula vacía.
1. Clínicamente la sospecha diagnóstica es:

a) Úlcera péptica perforada

b) Diverticulitis

c) Íleo metabólico

d) Trombosis mesentérica

e) Apendicitis complicada

2. Los estudios de laboratorio de mayor utilidad, son:

a) Química sanguínea y tiempos de coagulación

b) Química sanguínea y biometría hemátic

c) Glucosa y electrólitos séricos

d) Biometría hemática y glucosa

e) Biometría hemática y examen general de orina

3. El estudio de gabinete de primera elección es:

a) Tomografía axial computada

b) Ultrasonografía

c) Telerradiografía de tórax

d) Radiografías simples de abdomen

e) Serie esofagogastroduodenal
Antes de obtener los resultados de laboratorio, le realiza un dextrostix, el cual marca glucosa mayor de 400 mg/dL.
4. Con esta información, debe administrar prioritariamente:

a) Hipoglucemiantes orales

b) Insulina de acción intermedia

c) Insulina de acción prolongada

d) Líquidos parenterales

e) Se debe esperar el resultado de la glucemia central

5. El tratamiento definitivo consiste en:

a) Lisis de adherencias

b) Apendicectomía y lavado de cavidad

c) Resección intestinal

d) Billroth 1

e) Reposición hidroelectrolítica

51 CIRUGIA
Femenina de 43 años de edad. Originaria y residente del D.F., soltera, tabaquismo positivo ocasional, tres a cuatro cigarrillos por
semana desde los 18 años. Alcoholismo ocasional social. Antecedentes heredofamiliares: madre con insuficiencia venosa
periférica. Niega uso de anticonceptivos hormonales. Refiere dolor punzante en miembros inferiores desde los 16 años,
vespertino, que se ha exacerbado desde hace tres meses, sobre todo al permanecer de pie por tiempo prolongado. Edema
maleolar bilateral. A la exploración física: TA 110/70 mmHg, peso 56 kg, talla 1.58 m. FC 68 por minuto, FR 16 por minuto.
Extremidades inferiores con trayectos venosos visibles con dilataciones saculares en rosario en cara externa y posterior del
muslo izquierdo.
1. El diagnóstico más probable en este caso es:

a) Tromboflebitis

b) Tromboangitis obliterante

c) Várices esenciales

d) Livedo reticularis

e) Fenómeno de Raynaud

2. El método exploratorio básico en las enfermedades vasculares periféricas es:

a) Oscilometría

b) Palpación

c) Auscultación
d) Resistencia capilar

e) Tonometría

3. Un factor de riesgo para el desarrollo de trombosis venosa profunda es:

a) Puerperio

b) Tabaquismo

c) Traumatismo de cadera

d) Dislipidemia

e) Tratamiento con progestágenos

4. Una complicación de la trombosis venosa profunda es:

a) Infarto agudo del miocardio

b) Flegmasia cerulea dolens

c) Flegmasia alba dolens

d) Embolismo cerebral

e) Embolia pulmonar

5. Lesión dermatológica que con mayor frecuencia se asocia con la insuficiencia venosa de miembros inferiores:

a) Úlcera varicosa

b) Necrobiosis lipoidica

c) Flebodinia epidérmica

d) Erisipela

e) Eritema indurado

52 CIRUGIA
Masculino de 38 años. Antecedente de haber sufrido hace un año dolor tipo cólico en región lumbar derecha, con irradiación a
fosa iliaca derecha y testículo. Acude a la consulta por presentar náuseas y vómito, dolor tipo cólico en región lumbar izquierda,
irradiado a flanco izquierdo de tres días de evolución y que persiste a pesar del uso de múltiples analgésicos. Examen físico:
Inquieto con facies de dolor, tiene dolor a la palpación en región lumbar izquierda y en abdomen a nivel periumbilical del mismo
lado. Laboratorio: Hb 14 g/L, Hto 42%, leucocitos 13000 por mm3, urea 20 mg/dL, creatinina 1.2 mg/dL. Examen general de
orina con densidad de 1.028, eritrocitos abundantes, leucocitos 8 por campo, bacterias negativas. El ultrasonido renal muestra
pielocaliectasia izquierda moderada.
1. El diagnóstico del cuadro clínico descrito es:

a) Pielonefritis aguda

b) Glomerulonefritis aguda
c) Litiasis de vías urinarias

d) Tuberculosis urinaria

e) Síndrome nefrítico

2. El estudio que establece el diagnóstico de certeza es:

a) Ultrasonido renal

b) Placa simple de abdomen

c) Urografía excretora

d) Fluoroscopía

e) Urocultivo

3. Un germen causal frecuente de esta patología es:

a) Streptococcus

b) Chlamydia

c) Candida

d) Proteus

e) Gardnerella

4. Una complicación a largo plazo suele ser:

a) Hidronefrosis

b) Reflujo vesicoureteral

c) Prostatitis

d) Varicocele

e) Diverticulosis vesical

5. El tratamiento en el presente caso es:

a) Hiperhidratación y analgésicos

b) Antibióticos e hidratación

c) Ureteroscopía endoscópica

d) Laparotomía exploradora
e) Nefrostomía

53 GINECOOBSTETRICIA
Mujer de 22 años de edad. G: I, C: I. Cursó con embarazo de 37 semanas y ruptura prematura de membranas de 18 horas de
evolución. Cursando el tercer día de puerperio quirúrgico; se encuentra con ataque al estado general. Exploración física: TA
100/70 mm Hg, FC 92 por minuto, FR 22 por minuto, temperatura de 38.5ºC, Palidez de tegumentos, diaforética y regular estado
de hidratación. Campos pulmonares bien ventilados. El abdomen es doloroso a la palpación media y profunda, útero
subinvolucionado. Tacto vaginal: cérvix permeable en todo su trayecto, con abundantes loquios hemáticos y fétidos.
1. En esta entidad es común que:

a) La leucocitosis es un parámetro confiable para su diagnóstico

b) La fiebre se presenta en el primer día posparto

c) El dolor abdominal es el dato clínico más precoz

d) Se desarrolla en el segundo o tercer día del posparto

e) Se desarrolla dentro de las primeras 24 horas del posparto

2. Lo siguiente es correcto en esta entidad:

a) No es causa de esterilidad

b) La falta de higiene genital no es factor causal importante

c) La cesárea es el principal factor de riesgo

d) El parto precipitado en un factor de riesgo

e) El nivel socioeconómico no interviene en este proceso

3. Es el primer dato clínico presente en la mayoría de los casos:

a) Dolor abdominal

b) Fiebre

c) Útero subinvolucionado

d) Abundantes loquios

e) Loquios fétidos

4. Dentro de los gérmenes más comunes en esta entidad, se encuentran:

a) Estreptococos y bacteroides

b) Estafilococos y enterococos

c) E. coli y Gardnerella vaginalis


d) Micoplasma hominis y klebsiella

e) Peptococos y gonococos

.
5. El tratamiento antibiótico de elección para este caso es:

a) Ceftriaxona y gentamicina

b) Cefotaxima y gentamicina

c) Gentamicina y metronidazol

d) Ampicilina y gentamicina

e) Clindamicina y metronidazol

54 GINECOOBSTETRICIA
Mujer de 28 años de edad. Menarca a los 12 años, ciclos regulares, IVSA a los 19 años, varios compañeros sexuales, en
ocasiones utiliza preservativos. Cuadros de leucorrea en cuatro ocasiones tratados con diversos fármacos, no da más datos. Inicia
su sintomatología con leucorrea verde-amarillenta y abundante. Disuria y polaquiuria. A la especuloscopía: cérvix hiperémico,
leucorrea amarillenta. Al mover el espejo vaginal presenta salida de secreción mucopurulenta por uretra. Al tacto vaginal útero y
anexos sin alteraciones.
1. Con el cuadro clínico, la posibilidad diagnóstica es infección por:

a) Mycoplasma hominis

b) Trichomonas vaginalis

c) Gardnerella vaginalis

d) Neisseria gonorrhoeae

e) Chlamydia trachomatis

2. El periodo de incubación promedio de esta entidad es (en días):

a) 3 a 6

b) 8 a 10

c) 11 a 13

d) 14 a 16

e) >16

3. El agente etiológico de esta entidad tiene como característica:

a) Ser un microorganismo grampositivo

b) En histiocitos se observan cuerpos de Donovan


c) El mejor medio de cultivo es de Agar-chocolate

d) El mejor medio de cultivo es el de Thayer-Martin

e) La prueba más sensible es la detección del DNA mediante reacción en cadena de polimerasa

4. El tratamiento de elección en esta paciente es a base de:

a) Beta lactámicos

b) Cefalosporinas

c) Quinolonas

d) Aminoglucósidos

e) Tetraciclinas

5. Es la principal complicación de esta entidad:

a) Infección urinaria

b) Bartholinitis

c) Salpingitis

d) Inflamación anorrectal

e) Absceso anorrectal

55 GINECOOBSTETRICIA
Mujer de 49 años de edad. Hipertensa de 4 años de evolución, con control adecuado. AGO: Menarca a los 12 años, CM regulares
hasta hace 6 meses. MPF: OTB. DOC hace 1 año negativo a CaCu. GIII PIII. Refiere que desde hace 6 meses, presenta trastorno
en su menstruación, con ciclos menstruales de 20-30x 7-10, con abundante sangrado y expulsión de coágulos. El día de hoy,
acude a urgencias por mareos, cansancio, palpitaciones, además de hemorragia transvaginal de 3 días de evolución que no
corresponde a su periodo menstrual. FUM hace 10 días. Exploración física: bien hidratada, con palidez de tegumentos,
cardiopulmonar sin compromiso aparente, abdomen blando, depresible, no doloroso, se palpa tumoración a 4 cm por debajo de
cicatríz umbilical, consistencia dura, desplazable. A la especuloscopía: cérvix bien epitelizado, sangrado moderada cantidad,
proveniente de cavidad uterina. Al tacto vaginal: cerviz posterior, cerrado y formado, útero aumentado de tamaño de 16x12 cm,
irregular, fondos de saco libres. Resto de exploración sin alteraciones.
1. Respecto a esta entidad, es correcto que:

a) Es más frecuente en la menopausia

b) Son tumores exclusivos de la etapa reproductiva

c) Son tumores dependientes de progesterona

d) Es la neoplasia más frecuente del aparato genital femenino

e) Son tumores donde la malignización es muy frecuente

Se solicita ultrasonido pélvico y BH, con reporte de miomatosis de grandes elementos y ovarios no visibles y hemoglobina de 6
g/dL respectivamente.
2. De acuerdo a los resultados y los hallazgos clínicos, la conducta consiste en:

a) Manejo ambulatorio con hematínicos vía oral y programar cirugía

b) Manejo ambulatorio con hematínicos vía parenteral y programar cirugía

c) Manejo ambulatorio con hematínicos vía oral y análogos de la GnRH

d) Hospitalización, transfusión sanguínea y programar cirugía

e) Hospitalización, hematínicos vía parenteral y cirugía

3. La hemorragia irregular o continua, de intensidad variable, que hacen perder el carácter cíclico de la hemorragia menstrual
normal, se denomina:

a) Hipermenorrea

b) Metrorragia

c) Polimenorrea

d) Opsomenorrea

e) Polimenorragia

4. En cuanto a la cantidad, el trastorno menstrual que presenta la paciente, se denomina:

a) Hipermenorrea

b) Polimenorrea

c) Oligomenorrea

d) Metrorragia

e) Hipomenorea

5. De acuerdo a la localización de estos tumores, el que puede ocasionar mayor cantidad de sangrado es el:

a) Intramural

b) Cervical

c) Submucoso

d) Intraligamentario

e) Subseroso

56 GINECOOBSTETRICIA
Mujer de 38 años de edad. Hipertensa de 2 años, lleva tratamiento irregular al igual que el control prenatal. G: III, C: I, P: I.
Última cesárea hace 4 años, por sufrimiento fetal agudo. FUP hace 2 años, con antecedente de preeclampsia. Cursa con
embarazo de 37 semanas. Acude a urgencias por contracciones uterinas, una cada 20 minutos de 30 segundos de duración, desde
hace 6 hr. Percibe adecuados movimientos fetales. No refiere pérdidas transvaginales, ni sintomatología vasoespasmódica.
Exploración física: TA 130/80. FC 70 por minuto. FR 16 por minuto. Fondo uterino de 31 cm, PUVI, cefálico, FCF de 132 por
minuto, se palpa actividad uterina irregular. Al tacto vaginal: cérvix posterior con 3 cm de dilatación y 60 % de borramiento,
pelvis útil para eutocia. Ingresa a labor. Se toma registro cardiotocográfico, el cual es reactivo y se inicia conducción de TDP con
oxitocina 2 mUI por minuto. Al cabo de 2 h, la paciente refiere contracciones uterinas muy dolorosas, disminución de
movimientos fetales y salida de líquido transvaginal. Se revisa, encontrando útero hipertónico, FCF apenas audible, de 120 por
minuto, con descenso hasta 90 latidos por minuto. Al tacto vaginal: cérvix central con 5 cm dilatación y 80% borramiento,
hemorragia transvaginal, moderada cantidad, de color rojo oscuro.
1. Con los hallazgos clínicos, el diagnóstico es:

a) DPPNI

b) Vasa previa

c) Placenta previa

d) Polisistolia

e) Rotura uterina

2. Esta entidad se considera posible en el período comprendido:

a) Después de la semana 20 y 2 horas antes del inicio del trabajo de parto

b) Sólo en el tercer trimestre

c) Después de la semana 20 hasta antes del nacimiento del feto

d) Sólo durante el trabajo de parto

e) Desde el inicio del embarazo y el puerperio

3. El factor de riesgo más importante en la paciente para la aparición de esta entidad es:

a) Tabaquismo

b) Antecedente de cesárea

c) Edad materna

d) Multiparidad

e) Hipertensión arterial

4. En esta paciente, con feto vivo e hipertonía uterina, el manejo consiste en:

a) Suspender la oxitocina y dejar el TDP a libre evolución

b) Suspender la oxitocina y cesárea de urgencia

c) Solicitar ultrasonido de urgencia

d) Disminuir la dosis de oxitocina y dejar el TDP a libre evolución


e) Romper membranas amnióticas, para valorar líquido amniótica y descartar sufrimiento fetal

5. Complicación materna más frecuente que se puede presentar en este caso:

a) Rotura uterina

b) Coagulopatía por consumo

c) Acretismo placentario

d) Insuficiencia cardiaca congestiva

e) Hemorragia hepática

56 GINECOOBSTETRICIA
Mujer de 38 años de edad. Hipertensa de 2 años, lleva tratamiento irregular al igual que el control prenatal. G: III, C: I, P: I.
Última cesárea hace 4 años, por sufrimiento fetal agudo. FUP hace 2 años, con antecedente de preeclampsia. Cursa con
embarazo de 37 semanas. Acude a urgencias por contracciones uterinas, una cada 20 minutos de 30 segundos de duración, desde
hace 6 hr. Percibe adecuados movimientos fetales. No refiere pérdidas transvaginales, ni sintomatología vasoespasmódica.
Exploración física: TA 130/80. FC 70 por minuto. FR 16 por minuto. Fondo uterino de 31 cm, PUVI, cefálico, FCF de 132 por
minuto, se palpa actividad uterina irregular. Al tacto vaginal: cérvix posterior con 3 cm de dilatación y 60 % de borramiento,
pelvis útil para eutocia. Ingresa a labor. Se toma registro cardiotocográfico, el cual es reactivo y se inicia conducción de TDP con
oxitocina 2 mUI por minuto. Al cabo de 2 h, la paciente refiere contracciones uterinas muy dolorosas, disminución de
movimientos fetales y salida de líquido transvaginal. Se revisa, encontrando útero hipertónico, FCF apenas audible, de 120 por
minuto, con descenso hasta 90 latidos por minuto. Al tacto vaginal: cérvix central con 5 cm dilatación y 80% borramiento,
hemorragia transvaginal, moderada cantidad, de color rojo oscuro.
1. Con los hallazgos clínicos, el diagnóstico es:

a) DPPNI

b) Vasa previa

c) Placenta previa

d) Polisistolia

e) Rotura uterina

2. Esta entidad se considera posible en el período comprendido:

a) Después de la semana 20 y 2 horas antes del inicio del trabajo de parto

b) Sólo en el tercer trimestre

c) Después de la semana 20 hasta antes del nacimiento del feto

d) Sólo durante el trabajo de parto

e) Desde el inicio del embarazo y el puerperio

3. El factor de riesgo más importante en la paciente para la aparición de esta entidad es:

a) Tabaquismo
b) Antecedente de cesárea

c) Edad materna

d) Multiparidad

e) Hipertensión arterial

4. En esta paciente, con feto vivo e hipertonía uterina, el manejo consiste en:

a) Suspender la oxitocina y dejar el TDP a libre evolución

b) Suspender la oxitocina y cesárea de urgencia

c) Solicitar ultrasonido de urgencia

d) Disminuir la dosis de oxitocina y dejar el TDP a libre evolución

e) Romper membranas amnióticas, para valorar líquido amniótica y descartar sufrimiento fetal

5. Complicación materna más frecuente que se puede presentar en este caso:

a) Rotura uterina

b) Coagulopatía por consumo

c) Acretismo placentario

d) Insuficiencia cardiaca congestiva

e) Hemorragia hepática

58 GINECOOBSTETRICIA
Paciente de 31 años de edad. GII PI. FUP hace 3 años, producto masculino de 4100 gr obtenido con fórceps. Actualmente con
embarazo de 26 semanas. Acude a su segunda consulta de prenatal. Se refiere asintomática, percibe adecuados movimientos
fetales. Exploración fìsica: TA 110/80. Talla 1.62 m. Peso 83 kg. FU 28 cm, PUVI, FCF 145 por minuto, se difiere tacto vaginal,
extremidades sin edema. De sus resultados, sólo llama la atención la glucosa de 145 mg/dL. Por lo anterior, se indica prueba de
tamíz.
1. Esta prueba se recomienda realizarla preferentemente entre las semanas:

a) 12 a 15

b) 16 a 19

c) 20 a 23

d) 24 a 28

e) 29 a 32

2. Esta prueba se considera positiva, cuando la cifra es mayor de ( en mg/dL):


a) 120

b) 130

c) 140

d) 150

e) 160

3. La prueba de tamíz se considera diagnóstica de diabetes, en caso de una cifra mayor de glucosa de (en mg/dL):

a) 120

b) 140

c) 160

d) 180

e) 200

La prueba de tamíz resultó positiva, por lo que se realizò curva de tolerancia a la glucosa, con un valor alterado: 100 (basal), 205
( 1ª hora), 170 (2ª hora) y 150 (3ª hora).
4. Con los resultados anteriores, el diagnóstico es:

a) Diabetes Mellitus

b) Diabetes Gestacional

c) Intolerancia a los carbohidratos

d) Diabetes pregestacional

e) Diabetes química

5. La triada fundamental de riesgo en esta entidad la constituyen:

a) Antecedentes familiares de DM, obesidad y macrosomía fetal

b) Polihidramnios, obesidad y muerte fetal

c) Moniliasis recurrente, poliuria y polidipsia

d) Antecedente de diabetes gestacional, fetos malformados y obesidad

e) Abortos y glucosuria de repetición y antecedente de infertilidad

59 MEDICINA FAMILIAR
Femenina de 80 años de edad. Con diagnóstico de catarata senil, en protocolo de cirugía en fecha próxima. Presenta en forma
súbita edema palpebral, leve sensación de cuerpo extraño y lagrimeo, inicialmente en ojo derecho para que en el mismo día sea
en ambos. Nota además en ambas regiones preauriculares un nódulo de aproximadamente un centímetro de diámetro que a decir
de la señora cedieron el mismo día con masaje. Acude a la consulta externa el día siguiente con la sintomatología descrita, solo
se ha agregado, escasa secreción amarillenta. Recuerda que un familiar cercano presentó cuadro similar aunque menos intenso en
días previos. Exploración física: edema palpebral bilateral ++/+++, hiperemia de vasos conjuntivales superficiales, más intensa
en el fondo de saco conjuntival (fórnix), disminuyendo conforme nos acercamos al limbo esclerocorneal, quemosis y presencia
de folículos en la conjuntiva tarsal, con escasa secreción amarillenta en ambas hendiduras palpebrales.
1. Es correcto al respecto de la conjuntiva:

a) Membrana mucosa delgada y transparente que solo tapiza la superficie posterior de los párpados

b) Entre otras estructuras, recubre el segmento anterior del globo ocular hasta el limbo esclerocorneal

c) La porción palpebral esta adherida a la esclerótica y al limbo esclerocorneal

d) La porción bulbar esta adherida al tarso del párpado y a la órbita

e) La porción del fórnix que circunda el limbo esclerocorneal

2. Por las características descritas el tipo de ojo rojo presente corresponde a:

a) Inyección conjuntival

b) Inyección ciliar

c) Inyección periquerática

d) Inyección mixta

e) Equimosis subconjuntival

3. Con base en el cuadro clínico el diagnóstico más probable es:

a) Conjuntivitis hemorrágica aguda

b) Conjuntivitis por herpes simple

c) Conjuntivitis de inclusión del adulto

d) Conjuntivitis bacteriana primaria

e) Queratoconjuntivitis epidémica

4. El agente etiológico de esta entidad es:

a) Herpes simple

b) Adenovirus

c) Picornavirus

d) Staphylococcus epidermidis y aureus

e) Chlamydia trachomatis serotipos D-K

5. El tratamiento indicado en esta paciente es:


a) Eritromicina en gotas oftalmicas y Doxiciclina vía oral

b) Colirio de neomicina, gramicidina y bacitracina

c) Diclofenac y timolol en gotas oftálmicas

d) Aseo ocular con agua estéril, fomentos con agua fría, ciprofloxacina y esteroides oftálmicos

e) Aseo ocular con agua estéril, fomentos con agua fría y aciclovir oftálmico

60 MEDICINA FAMILIAR
Masculino de 46 años de edad, albañil y pintor. Con diagnóstico de diabetes mellitus desde hace un año Presenta desde hace tres
días, secundario a pintar área extensa de techo de oficina, dolor en hombro derecho hasta el tercio medio del brazo y el cuello,
además de limitación a la abducción y rotación. Que mejora con paracetamol tabletas. Se le solicita radiografía, comparativa de
hombros, sin encontrar alteraciones.
1. Con base en el cuadro clínico, la estructura lesionada es:

a) Ligamento coracoacriomial

b) Ligamento acromioclavicular

c) Tendón del manguito rotador

d) Tendón deltoideo

e) Bursa subacromial

2. Con base en el cuadro clínico el diagnóstico más probable es:

a) Tendinitis calcificada supraespinosa

b) Tendinitis bicipital

c) Lesión parcial del manguito rotador

d) Ruptura del manguito rotador

e) Hombro congelado

3. Es correcto al respecto de esta estructura anatómica:

a) Une el acromion y el apófisis coracoides

b) Une el músculo infraespinoso a la cabeza del húmero en una zona denominada canal bicipital

c) Es una bolsa llena de fluido situada entre la escápula y manguito de los rotadores

d) Emerge de la vaina sinovial de la articulación glenohumeral para continuar por la corredera bicipital del húmero

e) Es un tendón constituido por la unión de los músculos: supraespinoso, infraespinoso, redondo menor y subescapular

4. El estudio paraclínico de elección que se indica en este paciente es:


a) Potenciales evocados

b) Resonancia magnética nuclear

c) Radiografía en proyección lateral de la escápula

d) Radiografía en proyección anteroposterior con el paciente rotado a 45°

e) Ultrasonografía simple de hombro

5. El tratamiento indicado a este paciente es:

a) Reparación quirúrgica del ligamento

b) Infiltración de corticosteroides y anestésico en la bursa afectada

c) Remoción quirúrgica de la calcificación

d) Fisioterapia, antiinflamatorios no esteroides y el calor local

e) Reinserción del tendón del supraespinoso en la tuberosidad mayor del humero

61 MEDICINA FAMILIAR
Femenina de 71 años de edad, jubilada. Sin antecedentes heredofamiliares y personales de importancia (infecciones recientes,
traumatismos, afecciones dentales o déficit neurológico). Refiere dolor intenso, lascinante, de pocos segundos a minutos de duración,
que se originan habitualmente por masticar, hablar o sonreír, que se localiza en la región oral-auricular derecha. Exploración física: TA
140/85, sin lesiones cutáneas en cara, sin dolor a la palpación en región craneal, ni en zonas de proyección de senos paranasales
maxilares o frontales, conjuntivas de aspecto normal, pupilas isocoricas y normoreflexicas, apertura de boca sin limitación o dolor,
cavidad oral sin alteraciones gingivales o dentales aparentes. Cuello con arcos de movilidad adecuados no dolorosos. Al masticar se
provoca el dolor ya descrito, por lo que el paciente ya no coopera con el resto de la exploración.
1. Con base en el cuadro clínico el diagnóstico más probable es:

a) Trombosis del seno cavernoso

b) Neuralgia del trigémino

c) Síndrome de Wallenberg

d) Neurinoma del acústico

e) Neurinoma del ganglio de Gasser

2. Es correcto al respecto de esta entidad:

a) Es más común en el sexo masculino

b) Es más frecuente la afectación bilateral

c) La primera rama es la más afectada

d) En más del 80% la causa es esclerosis múltiple


e) Afecta principalmente a mayores de 50 años

3. La estructura anatómica afectada se caracteriza por qué:

a) Tiene cuatro ramas principales la temporal, oftálmica, maxilar y mandibular

b) Es el sexto par craneal

c) Emerge en la superficie medio-lateral de la protuberancia, con raíz sensitiva grande y motora pequeña

d) La rama mandibular solo es sensitiva

e) El núcleo sensorial es el más pequeño de los núcleos de los nervios craneanos

4. El tratamiento médico de elección en este paciente es:

a) Complejo B I.M. diario y prednisolona V.O.

b) Diclofenac y paracetamol tabletas

c) Acenocumarina y dipirona I.M.

d) Carbamazepina sola o asociada a fenitoína V.O.

e) Sólo se utiliza el tratamiento quirúrgico (rizotomía percutánea)

5. Es correcto al respecto del tratamiento indicado:

a) Tiene una respuesta favorable del 100%, 20% de recaídas y 50% de reacciones adversas

b) Son efectos secundarios poco frecuentes anemia aplásica y agranulocitosis

c) Se deben solicitar pruebas de función renal al inicio del tratamiento y después cada dos meses

d) Se puede presentar como complicación eventos tromboembólicos

e) Debe solicitarse como control durante el tratamiento TP y TPT

62 MEDICINA FAMILIAR
Femenina de 42 años de edad, secretaria y archivista. Se le diagnóstico diabetes mellitus desde hace ocho años y recibe como
tratamiento hipoglucemiantes orales (glibenclamida y metformina). Presenta desde hace diez meses dolor y parestesias en el
primero, segundo, tercer y porción radial del cuarto dedo de ambas manos, pero principalmente del lado derecho. El dolor en
ocasiones se irradia a antebrazo, hombro y cuello y se presenta por la noche. Cuando el dolor se intensifica el paciente realiza el
movimiento habitual para bajar la temperatura de un termómetro con los que refiere leve mejoría. El dolor usualmente aparece al
escribir y al conducir su automóvil. Exploración física: TA 110/70 mmHg, peso 72 kg, talla 1.62 m, el dolor se reproduce
primero, durante la hiperflexión de ambas muñecas aproximadamente después de dos minutos y segundo a la percusión sobre
zona de proyección del túnel del carpo.
1. Con base en el cuadro clínico el diagnóstico más probable es:

a) Tenosinovitis de Quervain

b) Distrofia simpático refleja (síndrome hombro-mano)


c) Radiculopatía cervical

d) Síndrome del túnel del carpo

e) Síndrome del escaleno anterior

2. Es correcto al respecto de esta entidad:

a) Es más frecuente en hombres

b) No existen padecimientos frecuentemente asociados a esta entidad

c) Es poco frecuente en los últimos meses del embarazo

d) Tiene relación con ciertas actividades laborales

e) El tratamiento es exclusivamente quirúrgico

3. La primera maniobra descrita en este paciente se conoce como de:

a) TInel

b) Phalen

c) Compresión carpal

d) Flick

e) Adson

4. La segunda maniobra descrita en este paciente se conoce como de:

a) Tinel

b) Phalen

c) Compresión carpal

d) Flick

e) Runge

5. La estructura anatómica principalmente afectada es:

a) Nervio mediano

b) Nervio radial

c) Extensor corto del pulgar

d) Extensores radiales del carpo


e) Vaina del abductor largo del pulgar

63 MEDICINA FAMILIAR
Femenina de 34 años de edad. Es intendente en escuela primaria. Presenta desde hace un año oscurecimiento, destrucción,
engrosamiento (hiperqueratosis) y alteración de la forma (distrofia), en la parte distal de las uñas de los pies. A la exploración
física: uñas de los pies despulidas, engrosadas, quebradizas y mancha blanquecino amarillenta en su porción distal y leves datos
inflamatorios en la piel circundante. Se diagnóstica a la paciente como portadora de onicomicosis. Se inició tratamiento con
ketoconazol el cual abandona un mes y medio después, ante la falta de respuesta.
1. Con base en el cuadro descrito la variedad clínica es:

a) Distal subungueal

b) Proximal subungueal

c) Blanco superficial

d) Por Cándida

e) Onicodistrofia total

2. Los agentes etiológicos que con mayor frecuencia están implicados en esta entidad son:

a) Trichophyton tonsurans y Mycrosporum canis

b) Trichophyton equinum y Trichophyton soudanense,

c) Trichophyton rubrum y Trichophyton mentagrophytes var. interdigitalis

d) Candida albicans y Candida tropicalis

e) Geotrichum candidum y Trichosporon cutaneum

3. Es correcto al respecto del ketoconazol:

a) Su biodisponibilidad no depende de la acidez gástrica

b) Su vida media no es dosis dependiente

c) Es extensamente metabolizado a nivel renal

d) El tratamiento se indica por espacio de un mes

e) Entre sus inconvenientes son: su absorción errática y la toxicidad hepática

4. Se inicia tratamiento con itraconazol, es correcto al respecto de este fármaco:

a) El efecto terapéutico solo persiste una semana después de terminado en tratamiento

b) Produce cura micológica en el 50% de los casos, y 2% de recaídas

c) No se ha informado interacciones con otros fármacos


d) Actualmente disponemos de dos pautas de tratamiento con eficacia similar

e) Los efectos adversos más frecuentes son hematológicos y renales

5. Otra característica de este fármaco es:

a) Sus vías de administración son enteral y parenteral

b) Se absorbe a nivel del tracto gastrointestinal y se metaboliza en el riñón

c) Es lipofílico y queratinofílico, con gran afinidad por piel, mucosas y uñas

d) Su principal vía de excreción es la hepática

e) Actúa inhibiendo la síntesis del ergosterol, lo hacen a nivel de la enzima escualenoperoxidasa

64 MEDICINA FAMILIAR
Masculino de siete años de edad, el cual durante revisión médica escolar, se le encuentra con peso bajo. La maestra lo refiere
como distraído, poco participativo y con tendencia al sueño. Por este motivo es traído a la consulta, se menciona que desde hace
seis meses presenta: hiporexia, dolor abdominal tipo cólico frecuente, posprandial y en ocasiones evacuaciones disminuidas de
consistencia que remiten en forma espontánea además de cefalea y palidez. Exploración física: peso 17 kg, talla 105.5 cm,
palidez + de tegumentos, distensión abdominal. Se solicita coproparasitoscópico en serie de tres y se informa la presencia de
Giardia lamblia.
1. Es correcto al respecto del parásito intestinal, detectado en el paciente, que:

a) Se localiza habitualmente en la región cecal del intestino grueso

b) En su porción media tiene estructuras llamadas proglótides

c) El escólex posee ventosas y una hilera de ganchos, vive en las últimas porciones del íleon

d) Tiene forma de pera. con seis flagelos y un disco succionador en su parte ventral

e) El macho mide 3 a 5 mm y la hembra de 9 a 12 mm de longitud

2. El hábitat natural en el ser humano es:

a) Duodeno y yeyuno

b) Últimas porciones del íleon

c) Asa sigmoides y colon ascendente

d) Región cecal de intestino grueso

e) Colédoco y vesícula biliar

3. En cuanto a la patogenia de esta parasitosis es correcto que:

a) Las lesiones iniciales son úlceras pequeñas que se extienden hasta la muscularis mocosae

b) Producen úlceras profundas en forma de “botón de camisa”


c) Al paso por la vía respiratoria baja producen focos mínimos de neumonía o síndrome de Löeffler”

d) Deposita sus huevos en las márgenes del ano

e) Provoca por efecto mecánico atrofia de las microvellosidades

4. El tratamiento de elección indicado al paciente es:

a) Citrato de piperacina

b) Pamoato de pirantel

c) Metronidazol

d) Mebendazol

e) Albendazol

5. El tratamiento alternativo es:

a) Furazolidona o tinidazol

b) Tinidazol o prazicuantel

c) Hidroxinaftoato de befenio o pamoato de pirantel

d) Mebendazol o albendazol

e) Diyodohidroxiquinoleína y tinidazol

65 MEDICINA INTERNA
Femenina de 64 años de edad. Tiene antecedentes de ser hipertensa de 15 años de evolución, tratada con captopril, además de
padecer gastritis crónica tratada con ranitidina. Presenta dolor localizado en rodilla derecha, de tres meses de evolución,
inicialmente leve, posteriormente moderado, aumenta con el ejercicio y cede con el reposo, se asocia con limitación para el
movimiento. A la exploración física: obesa, talla 1.60 m, peso 73 kg; rodilla derecha con crepitación dolorosa a la movilización.
No hay derrame articular. Las radiografías mostraron disminución del espacio articular, esclerosis subcondral y osteofitos.
Laboratorio: ácido úrico 5 mg/dL, factor reumatoide negativo, biometría hemática normal.
1. En la fisiopatología de la enfermedad articular que padece la paciente interviene:

a) Disminución de líquido sinovial

b) Presencia de pirofosfato cálcico en líquido articular

c) Degeneración exagerada del cartílago articular

d) Artritis autoinmunitaria

e) Traumatismo directo intenso

2. El estudio de mayor utilidad para detectar alteraciones en este caso es:

a) Radiografía simple
b) Resonancia magnética nuclear

c) Ultrasonido

d) Densitometría

e) Artroscopia

3. El tratamiento farmacológico inicial que indica es:

a) Prednisona

b) Azatioprina

c) Dextropropoxifeno

d) Glucosamina

e) Acetaminofén

4. El objetivo del tratamiento es:

a) Inhibir la degradación del cartílago

b) Inmovilizar la articulación

c) Disminuir la inflamación sinovial

d) Disminuir la síntesis de ácido úrico

e) Abatir la respuesta autoinmunitaria

Usted realiza punción articular para análisis de líquido sinovial, el cual se informa con ausencia de cristales, cultivo negativo,
leucocitos menores de 2 000/mm3 polimorfonucleares menores de 25%.
5. La enfermedad con la que tiene que hacer diagnóstico diferencial es:

a) Tumor sinovial

b) Síndrome de Reiter

c) Artritis traumática

d) Artritis psoriásica

e) Artritis reumatoide

66 MEDICINA INTERNA
Masculino de 50 años de edad. Antecedente de etilismo crónico, agudizado desde los 25 años llegando a la embriaguez cada mes.
Toma sucralfato irregularmente desde hace seis meses por gastritis antral, reflujo biliar y “erosión” duodenal. Padecimiento actual
de 24 horas de evolución con cuatro evacuaciones melénicas fétidas, acompañadas de dolor epigástrico intenso, transfictivo que se
agudiza durante el posprandio inmediato, astenia, adinamia y anorexia. Se encuentra pálido (++) llenado capilar 8 segundos,
signos vitales: TA en decúbito 110-50 mmHg, TA de pie 120/60 mmHg, FC 120 por minuto, FR 20 por minuto, temperatura
35.5°C, dolor intenso en epigastrio, hiperestesia, sin datos de irritación peritoneal, Hb 10g/dL, Hto 35%.
1. En este caso, la causa más probable de hemorragia es:

a) Esofagitis erosiva

b) Várices esofagogástricas rotas o laceradas

c) Gastritis congestiva

d) Úlcera duodenal

e) Úlcera gástrica

2. El dato endoscópico que indica alto riesgo de recidiva de hemorragia es:

a) Vaso visible sin hemorragia

b) Coágulo adherido a úlcera

c) Restos de hematina

d) Hemorragia “en capa”

e) Úlcera “limpia”

3. El dato endoscópico que establece la indicación para tratamiento esclerosante es:

a) Úlcera “limpia”

b) Úlcera de 2.5 cm

c) Úlcera de más de 3 cm

d) Vaso visible sin hemorragia

e) Restos de hematina

4. El fármaco que reduce la secreción ácida gástrica y el flujo esplácnico es:

a) Omeprazol

b) Lanzoprazol

c) Somatomedina

d) Somatostatina

e) Ranitidina

5. La complicación que ocupa por frecuencia el segundo lugar en este padecimiento es:

a) Obstrucción intestinal

b) Estenosis
c) Penetración

d) Malignización

e) Perforación

67 MEDICINA INTERNA
Femenina de 43 años de edad. Acude a consulta por edema palpebral y de ambos tobillos de dos semanas de evolución, así como
palidez generalizada, astenia, adinamia e hiporexia, cuando acude a miccionar refiere abundante espuma y cuatro días previos
con acúfeno, fosfeno y cefalea universal.
1. En este caso la primera acción clínica a efectuar es:

a) Medición de la presión arterial y EKG

b) Examen de fondo de ojo

c) Verificar si existe edema e hipertensión arterial

d) Investigar antecedente de insuficiencia renal

e) Solicitar valoración nefrológica

2. En caso de haber encontrado alteraciones en la presión arterial, los siguientes estudios paraclínicos a solicitar son:

a) Cotecolaminas en orina

b) Determinación de ácido vanilmandélico en orina

c) Telerradiografía de tórax y ultrasonido renal

d) Depuración de creatinina en orina de 24 horas

e) Examen general de orina y perfil de lípidos

Los hallazgos indican TA 140/100 mmHg, colesterol 385 mg/dL, triglicéridos 312 mg/dL, ultrasonido renal normal,
gammagrama renal con disminución de la perfusión en ambos riñones en 12% y 8% para el riñón derecho e izquierdo
respectivamente. En las pruebas de función hepática sólo TGO 38 UI y DHL 415 UI, albúmina 3.1 mg/dL, en el EGO: cilindros
grasos, proteínas ++, cililndros hialinos y pH 6.5, catecolaminas 50 microgramos/día; ácido vanilmandélico 55 mg/día,
depuración de creatinina 80 mL/min, proteinuria 5.8 g/24 horas.
3. El diagnóstico que establece es:

a) Síndrome nefrítico

b) Síndrome nefrótico

c) Hipertensión primaria

d) Hipertensión secundaria

e) Nefropatía hipertensiva

4. Previo al inicio del tratamiento es importante establecer la etiología y en el grupo de edad de la paciente, la más frecuente es
glomerulonefritis:
a) Posestreptocócica

b) Membranosa

c) Hipertensión esencial

d) Hipertensión secundaria

e) De cambios mínimos

5. El tratamiento de primera elección en este caso es:

a) Ciclofosfamida

b) Ciclosporina

c) Azatioprina

d) Prednisona

e) Furosemida

68 MEDICINA INTERNA
Femenina de 56 años de edad. Comerciante. Tabaquismo intenso durante 30 años a razón de 20 cigarrillos por día. Se conoce
diabética de 12 años de evolución, controlada actualmente con insulina de acción intermedia; tiene insuficiencia renal crónica
con tratamiento sustitutivo con diálisis peritoneal continua ambulatoria. Refiere parestesias en miembros pélvicos desde hace
cuatro años, utiliza bastón para ayudarse a deambular. Acude a consulta por secreción purulenta en pie derecho a nivel de
articulación metatarsofalángica de primer ortejo. A la exploración física se aprecia zona de hiperqueratosis y eritema perilesional
con úlcera de aproximadamente dos centímetros de diámetro con abundante salida de material purulento.
1. La causa más frecuente de la aparición de lesiones de este tipo es por:

a) Distribución anormal del peso corporal

b) Insuficiencia venosa

c) Émbolos en miembros pélvicos

d) Tabaquismo intenso

e) Estasis sanguínea

2. El estudio radiográfico de la lesión demostró datos de osteomielitis. Un estudio que contribuye para normar conducta
terapéutica en este caso es:

a) Angiografía selectiva

b) Gammagrama

c) Ultrasonido Doppler

d) Cultivo de exudado del pie


e) Tomografía axial de pie

3. El estudio anterior demuestra que existe aún adecuada circulación. El manejo que se recomienda es:

a) Desbridación amplia

b) Desarticulación del ortejo

c) Administración intraarterial de antibióticos

d) Administración parenteral de antibióticos

e) Administración parenteral de antibióticos y resección del sitio afectado

4. El tipo de gérmenes que se encuentran con mayor frecuencia en estos casos es:

a) Grampositivos

b) Gramnegativos

c) Anaerobios

d) Flora mixta

e) Hongos

5. Una vez resuelto el proceso infeccioso usted recomienda para prevenir nueva formación de úlceras:

a) Medidas de higiene venosa

b) Cuidados de la mecánica de los pies

c) Antiagregantes plaquetarios

d) Vasodilatadores periféricos

e) Uso de medias elásticas

69 MEDICINA INTERNA
Masculino de 38 años de edad. Antecedente de hipertensión arterial desde hace tres años. Padecimiento de tres días de
evolución, inició con disnea rápidamente progresiva, edema de miembros inferiores y cefalea intensa holocraneana. La TA es
180/130 mmHg. El estudio de fondo de ojo mostró papiledema bilateral, con hemorragias en flama. Tenía ingurgitación yugular
grado II, estertores crepitantes en ambas bases pulmonares y ritmo de galope. El EGO con proteínas +++ y cilindros hialinos.
En el EKG se observan cambios inespecíficos del segmento ST. En la radiografía de tórax, hay datos de hipertensión
venocapilar pulmonar.
1. La hipertensión arterial en este paciente se debe más probablemente a un aumento en la producción de:

a) Catecolaminas

b) Renina

c) Aldosterona
d) Angiotensina

e) Ácido vanilmandélico

2. Las alteraciones en el fondo de ojo se deben a hipertensión arterial:

a) Esencial sin control

b) Reciente poco activa

c) Reciente muy activa

d) Crónica muy activa

e) Crónica activa

3. El estudio más útil para el diagnóstico de la causa de la hipertensión arterial en este caso es la medición de:

a) Actividad de la renina plasmática

b) Catecolaminas urinarias

c) 17 -cetoesteroides en orina

d) Aldosterona en suero

e) Metabolitos del ácido araquidónico

4. El tratamiento inmediato de elección en este paciente debe realizarse con:

a) Captopril

b) Isosorbide

c) Nifedipino

d) Diazóxido

e) Nitroprusiato de sodio

5. El tratamiento definitivo debe incluir de manera primordial:

a) Plastía de arteria renal

b) Resección de adenoma suprarrenal

c) Resección de tumoración intracraneal

d) Resección de feocromocitoma

e) Radiación de la hipófisis
70 MEDICINA INTERNA
Femenina de 33 años de edad. Antecedentes de crisis convulsivas desde los siete años, bajo tratamiento con carbamazepina en
dosis de 200 mg al día, quien presenta dos días previos un número no determinado de convulsiones tónico-clónicas, motivo por
el que es traída a urgencias. A la exploración física se encuentra TA 110/70 mmHg, FC 100 por minuto, FR 30 por minuto,
temperatura 36°C. Somnolienta, con respiración superficial entre las crisis, pupilas isocóricas con reflejos normales. En tórax
sibilancias generalizadas sin estertores; abdomen globoso sin anormalidades. Extremidades con reflejos osteotendinosos
normales. Los exámenes de laboratorio informan: Hb 15.1 g/dL, Hto 31.5%, leucocitos 6 050/mm3, glucosa 101 mg/dL,
creatinina 0.8 mg/dL, Na 144 mmol/L, K 4.4 mmol/L, ALT 33 UI/L, AST 47 UI/L, TP 10.9/11.9/seg, TPT 28/seg, plaquetas 250
000/mm3, pH arterial 7.50, paO2 90.8 mmHg, paCO2 21.4 mmHg, HCO3 17 mEq/L, CO2 T 17.7 mEq/L, Sat. 98% (FiO2 0.21).
Nivel de carbamazepina en sangre 2.02 mg/dL (normal 4.0 a 10.0 mg/dL). Radiografía de tórax con pequeñas opacidades
diseminadas en la base pulmonar derecha.
1. El tipo de crisis que presenta la paciente corresponde a una:

a) Parcial simple

b) Sintomática

c) Criptogénica

d) Idiopática

e) Relacionada a la situación

2. La causa más probable de desencadenamiento de las crisis convulsivas en esta paciente es:

a) Infección pulmonar

b) Broncospasmo

c) Concentración sérica inadecuada de anticonvulsivante

d) Crisis convulsiva psicógena

e) Hiperventilación

La paciente recibió tratamiento con diacepam 20 mg IV en una hora, previa intubación traqueal y persistió con crisis convulsivas
de iguales características.
3. La mejor conducta terapéutica en este momento es administrar:

a) Carbamazepina

b) Fenitoína

c) Clonacepam

d) Primidona

e) Ácido valproico

4. La causa de la falta de respuesta al fármaco administrado es:

a) Elección inadecuada

b) Dosis inadecuada
c) Interacción medicamentosa carbamazepina-diacepam

d) Falta de corrección de trastorno metabólico

e) Presencia de status epilepticus

5. El porcentaje de mortalidad por la complicación que presenta la paciente es de:

a) 0 a 9

b) 10 a 20

c) 21 a 39

d) 40 a 50

e) > de 50

71 PEDIATRIA
Femenina de 8 años de edad. Es llevada por los padres a consulta para la expedición de un certificado de salud, ya que desea
ingresar al centro deportivo de su localidad. Al revisar la Cartilla Nacional de Vacunación usted encuentra como le aplicaron las
vacunas: Al nacer: BCG, a los dos meses: Sabin y DPT, a los cuatro meses: Sabin y DPT, a los doce meses: antisarampionosa.
1. En cuanto a las recomendaciones generales de vacunación de esta paciente usted le menciona a los padres lo siguiente:

a) Es necesario administrar la segunda dosis de vacuna BCG, ya que no fue administrada a los seis años

b) Se debe aplicar por lo menos una dosis más de vacuna contra la poliomielitis, vacuna Sabin

c) Se requiere completar el esquema con vacuna DPT ya que falta una dosis a los 6 meses y los refuerzos

d) Es necesario aplicar una dosis de vacuna contra sarampión, rubéola,

e) Se debe reiniciar el esquema de vacunación como si no hubiera recibido ninguna vacuna.

2. En relación a la protección contra difteria, tosferina y tétanos, usted indica:

a) No aplicar más dosis de DPT de células completas por el riesgo de efectos adversos graves

b) No aplicar más dosis de DPT, ya que con dos dosis se alcanza una protección cercana al 100%

c) Completar el esquema únicamente para tétanos (aplicación de toxoide tetánico)

d) Aplicar la tercera dosis para completar el esquema primario

e) Aplicar una dosis de DPT más antihaemophilus influenzae

3. En relación a la protección contra hepatitis B, ¿Qué recomendaciones le haría a los padres?

a) No es posible vacunarlo porque viene combinada en la vacuna pentavalente (DPT-HiB-HB)

b) Debe recibir la vacuna contra hepatitis B a la brevedad posible


c) Es innecesaria la vacuna pues es un sujeto que no ha recibido ninguna transfusión

d) Es innecesario vacunarlo, ya que esta vacuna se aplica a los 2, 4 y 6 meses de edad (vacuna pentavalente)

e) Hay que investigar la competencia de su sistema inmune por ser una vacuna de virus atenuados

4. En relación con la vacuna contra Haemophilus influenzae tipo b usted les informa a los padres que:

a) Debe recibir la vacuna combinada con DPT

b) Debe recibir la vacuna independientemente de la edad

c) Es innecesario aplicar la vacuna en individuos sanos mayores de 5 años de edad

d) Es innecesaria la vacuna por no estar incluida en la Cartilla Nacional de Vacunación

e) Es innecesaria la inmunización por tratarse de un microorganismo poco frecuente en nuestro país

5. En cuanto a la vacunación contra sarampión, rubéola y parotiditis se recomienda:

a) Realizar exámenes serológicos contra estos virus antes de vacunar

b) Dejar el esquema de vacunación sin cambios

c) Aplicar solamente vacuna contra rubéola

d) Aplicar únicamente vacuna contra rubéola y parotiditis

e) Vacunar con triple viral (sarampión-rubéola-parotiditis)

72 PEDIATRIA
Masculino de siete meses de edad. Es llevado a consulta por su madre quien refiere que desde hace tres días presenta diarrea con
moco y sangre, en número de 8 evacuaciones al día, así como fiebre de 38°C. Niega la presencia de vómitos, así como alguna
otra sintomatología. Exploración física: cráneo normal, fontanela anterior hundida, ojos hundidos con llanto sin lágrimas, boca y
lengua secas con saliva espesa. Cardiopulmonar sin alteraciones. Abdomen normal con peristalsis ligeramente aumentada,
genitales y extremidades normales, la madre refiere que ha notado la presencia de sed aumentada.
1. El diagnóstico más probable de este paciente es:

a) Invaginación intestinal

b) Estrías anales

c) Intolerancia a las proteínas de la leche

d) Disentería

e) Evacuaciones rojas por algún alimento

2. La etiología más frecuente de este padecimiento es:

a) Empleo de anti-motílicos
b) Constipación intestinal crónica

c) Reacción mediada por IgE

d) Infección por bacterias

e) Ingesta de colorantes en alimentos

3. El diagnóstico en este tipo de pacientes se realiza mediante:

a) Tacto rectal (signo grosella)

b) Visualización directa

c) RAST positivo y elevación de IgE

d) Antecedente de ingesta alimentaria

e) Coprocultivo

4. Complicación característica de esta enfermedad:

a) Perforación intestinal

b) Rectorragia

c) Deshidratación

d) Anemia

e) Intoxicación por colorantes

5. El tratamiento de dicho padecimiento es:

a) Colon por enema

b) Dieta laxante y líquidos

c) Cambio de fórmula láctea

d) Inicio de antimicrobiano e hidratación

e) Suspender el alimento con el colorante

73 PEDIATRIA
Femenina de 14 años de edad, quien acude por presentar un padecimiento de seis meses de evolución, caracterizado por perdida
de peso de 3 kg, astenia y adinamia. Desde hace cuatro meses refiere caída de cabello y enrojecimiento intenso a nivel de
pómulos, cuando se encuentra en lugares al aire libre. Desde hace un mes con aumento de volumen y temperatura en rodilla
izquierda. Exploración física con peso y talla en la percentil 25, llama la atención la presencia de eritema malar, así como la
presencia de palidez de tegumentos. A nivel de mucosa oral, se observa la presencia de úlcera en paladar. En tórax se ausculta
disminución de los ruidos cardiacos, siendo estos rítmicos y sin fenómenos agregados, Abdomen y genitales normales. La rodilla
izquierda con aumento de volumen de 3 cm. En comparación con la derecha, aumento de la temperatura local y limitación a la
extensión.
1. El diagnóstico más probable de esta paciente es:

a) Fiebre reumática

b) Artritis idiopática juvenil

c) Lupus eritematosos sistémico

d) Artritis reactiva

e) Dermatomiositis juvenil

2. Los estudios de laboratorio que solicita son:

a) Antiestreptolisinas, exudado faríngeo

b) Anticuerpos antinucleares, anti-DNA

c) Factor reumatoide, anticuerpos antinucleares

d) Velocidad de sedimentación globular, proteína C reactiva

e) Enzimas musculares, fosfatasa alcalina

3. El manejo de esta paciente incluye:

a) Penicilina benzatina mensual, acido acetilsalicílico

b) Metotrexate, naproxeno

c) Filtro solar, prednisona, azatioprina

d) Antiinflamatorio no esteroideo, antibioticoterapia

e) Prednisona, terapia de rehabilitación

4. Las complicaciones más comunes en este padecimiento son:

a) Valvulopatía reumática

b) Uveítis anterior, amiloidosis

c) Calcinosis, hipertensión arterial pulmonar

d) Nefropatía, hemorragia pulmonar

e) Remisión espontánea

5. La mortalidad en este caso, esta asociada comúnmente a:

a) Infección e inmunosupresión

b) Hemorragia pulmonar
c) Cardiopatía

d) Daño renal

e) Sepsis

74 PEDIATRIA
Lactante de 8 meses de edad. Presenta llanto persistente, inconsolable al cuidado de la madre, distensión abdominal +++ y
vómito de contenido gastrobiliar, tiene antecedentes de presentar evacuaciones diarreicas desde hace 5 días, acompañándose de
cuadro febril. A la EF en abdomen se palpa una zona indurada como “morcilla”, distensión, disminución de la peristalsis y dolor
a la palpación. Ha sido manejado con soluciones parenterales así como trimetoprim con sulfametoxazol.
1. Con los datos anteriores, que complicación espera encontrar:

a) Íleo metabólico

b) Neumatosis intestinal

c) Peritonitis sin perforación

d) Invaginación intestinal

e) Isquemia intestinal

2. Es un dato específico de esta complicación del síndrome diarreico:

a) Acidosis metabólica, hiponatremia, hipokalemia

b) Evacuaciones sanguinolentas

c) Distensión de asas intestinales

d) Ausencia de peristalsis

e) Presencia de líquido intraabdominal

3. El diagnóstico específico por imagenología se realiza con:

a) Radiografía simple de abdomen

b) Colon por enema

c) Ecografía

d) Tomografía

e) Serie gastrointestinal

4. El sitio de lesión anatomoclínica más frecuente de esta entidad es:

a) Colo-cólica

b) Ceco-cólica
c) Ileo-ileal

d) Ileo-cecal

e) Ileo-cólica

5. El pronóstico de esta entidad es:

a) Bueno para la vida y la función

b) Bueno para la vida, malo para la función

c) Reservado

d) Malo a corto plazo

e) Con secuelas a mediano plazo

75 PEDIATRIA
Lactante menor de 9 meses de edad. Es traído a la consulta por sus padres, por presentar dermatosis de un mes de evolución en
región glútea y periné caracterizado por eritema, erosiones y costras serohemáticas, refiere tratamiento a base de miconazol
tópico presentando ligera mejoría.
1. Con los datos anteriores, ¿cuál es su diagnóstico?

a) Candidosis

b) Psoriasis

c) Dermatitis seborreica

d) Dermatitis de contacto

e) Dermatitis de pañal

¿Cuál es la zona más afectada en esta enfermedad?

a) Articulaciones en la zona de extensión

b) Cuero cabelludo

c) Manos y antebrazos

d) Genitales, periné y nalgas

e) Pliegues y área del pañal

3. ¿Cuál es el agente etiológico?

a) Pseudomonas

b) Enterococos

c) Candida albicans
d) Gramnegativos

e) Estafilococo dorado

4. ¿Qué factores intervienen en el desarrollo de este padecimiento?:

a) Alergénicos por los componentes del pañal

b) Mecanismos de autoinmunidad

c) Sensibilización, hidratación excesiva y amoniaco

d) Sensibilización por urea y creatinina

e) Producción de exotoxinas por el agente etiológico

5. El tratamiento de dicho padecimiento es:

a) Gentamicina tópica

b) Miconazol en crema

c) Fluocinolona

d) Baños de asiento con ácido acético

e) Aceite de almendras dulces

76 PEDIATRIA
Masculino de 6 años de edad. Acude a consulta externa por presentar tos seca de predominio nocturno y matutino de 5 meses de
evolución, ocasionalmente se asocia con datos de dificultad respiratoria, dolor torácico. Dentro de los antecedentes de
importancia ha cursado con infecciones de vías respiratorias recurrentes, que se acompañan de tos de predominio nocturno, de
repetición desde los 3 años de edad. Exploración física: peso 18 kg, frecuencia respiratoria 25 por minuto, temperatura 37°C,
ruidos cardiacos 88 por minuto, conjuntiva tarsal granulosa, mucosa nasal pálida, con puentes hialinos, orofaringe con amígdalas
grado III, crípticas, cuello con adenopatías de 1 cm. Móviles, dolorosas a la palpación, área cardiopulmonar con ruidos cardiacos
rítmicos de buena intensidad, campos pulmonares con estertores gruesos y silbantes, abdomen blando con peristaltismo presente,
piel generalizada seca.
1. Con los datos anteriores, ¿qué diagnóstico integra?

a) Neumonía

b) Bronconeumonía

c) Tuberculosis

d) Asma

e) Enfermedad por reflujo

2. Para confirmar su diagnóstico, ¿qué estudio solicita?

a) Radiografía de tórax
b) Broncoscopía

c) BAAR con expectoración

d) Espirometría

e) Serie esofagogastroduodenal

3. El antecedente de importancia en éste caso es:

a) Atopia

b) Inmunodeficiencia

c) Coombe positivo

d) Infección de vías superiores recurrentes

e) Prematurez

4. El tratamiento se basa en:

a) Antibióticos y fisioterapia

b) Antibióticos y expectorantes

c) Antifímicos

d) Antiácidos

e) Esteroides locales y broncodilatadores

5. La siguiente aseveración es correcta en esta patología:

a) Es la enfermedad crónica más frecuente en pediatría

b) La prevalencia es de 5%

c) No es frecuente en pediatría

d) Causa gran mortalidad

e) Favorece cardiopatía

77 PEDIATRIA
Lactante masculino de 4 años de edad. Es traído a consulta por presentar evacuaciones aguadas de color amarillento en número
de tres durante la mañana, acompañadas de dolor abdominal de tipo cólico, ha vomitado en una sola ocasión de contenido
alimentario desde hace una semana, la madre refiere que el niño no quiere comer bien, este cuadro lo ha presentando varias
veces durante los últimos 6 meses y que en ocasiones sufre de estreñimiento. Exploración física: Peso 14 500 g, talla 96 cm, FC
110 por minuto, FR 28 por minuto, temperatura 36.5 °C, se aprecia lactante adelgazado, activo, reactivo, con palidez de piel y
tegumentos, sin datos de deshidratación, destaca abdomen con discreta distensión, peristalsis discretamente aumentada, se
despierta dolor en región del epigastrio, genitales y resto de la exploración sin alteraciones aparentes.
1. Con los datos anteriores, el diagnóstico más probable es :
a) Gastroenteritis probablemente viral

b) Gastroenteritis probablemente bacteriana

c) Intolerancia a la lactosa

d) Síndrome de absorción intestinal deficiente

e) Parasitosis intestinal

2. El agente etiológico más probable en este caso es:

a) Giardia lamblia

b) Shigella

c) Lactosa

d) Salmonella sp.

e) Rotavirus

3. El estudio más indicado para confirmar su diagnóstico de sospecha es:

a) Rotatest y ELISA

b) Coproparasitoscópico y líquido duodenal

c) Prueba de Sereny

d) pH y sustancias reductoras en heces

e) Coprocultivo

4. El tratamiento específico de primera elección para esta patología es:

a) Rehidratación

b) Trimetoprim-sulfametoxazol a 8 mg/kg/día

c) Metronidazol 20 mg/kg/día

d) Apoyo nutricional y dieta astringente

e) Furazolidona 7mg/kg/día

5. ¿Qué complicación puede presentar el paciente?

a) Protrusión anal

b) Mala absorción
c) Perforación intestinal

d) Invaginación intestinal

e) Sangrado de tubo digestivo bajo

78 PEDIATRIA
Recién nacido de 4 días de vida. Es llevado a consulta por la presencia de tinte ictérico. Antecedente de ser producto de gesta I,
madre de 18 años de edad, obtenido por parto eutocico, Apgar 8/9, Silverman 0/0, peso 3 000 gramos, al nacimiento, el cual fue
egresado del hospital con su madre. Ha sido alimentado al pecho materno. La madre refiere que hace 24 horas nota la presencia
de color amarillo en la piel. Exploración física: Se encuentra reactivo, FC 130 por minuto, FR 40 por minuto, temperatura
36.8ºC, reflejos primarios presentes, normales tinte ictérico moderado, cardiopulmonar sin compromiso, abdomen sin
hepatoesplenomegalia. Resto negativo. Laboratorio: BH normal, binomio O Rh+, Coombs negativo, bilirrubina directa 0.4
mg/dL, bilirrubina indirecta 12.5 mg/dL.
1. Con los datos anteriores, su impresión diagnóstica es:

a) Ictericia por alimentación con leche materna

b) Hepatitis

c) Isoinmunización materno fetal

d) Ictericia fisiológica

e) Esferocitosis congénita

2. De acuerdo a su diagnóstico, el tratamiento de elección es:

a) Fototerapia

b) Exanguineotransfusión

c) Administración de fenobarbital

d) Repetir paraclínicos

e) No requiere de manejo específico

3. Uno de los criterios para descartar esta entidad es:

a) Bilirrubinas totales menores de 12.9 mg

b) Coombs negativo

c) Aparición de la ictericia en las primeras 24 horas

d) Ictericia entre el 2do y 3er día de vida

e) Formula roja normal

4. La mayor cantidad de bilirrubina proviene del catabolismo de la proteína:

a) Globina
b) Hem

c) Y - Z

d) Glucoronil transferasa

e) Albúmina

5. La complicación más grave de la hiperbilirrubinemia es:

a) Encefalopatía hiperbilirrubinémica

b) Acidosis

c) Hipoacusia

d) Retardo en el crecimiento

e) Choque

79 URGENCIAS
Paciente masculino de 60 años de edad, obeso, sedentario. Tiene como antecedente de importancia ser portador de diabetes
mellitus de cinco años de evolución controlado con glipizida 10 mg V.O. c/24 horas. Tabaquismo positivo desde los 18 años a
razón de cinco cigarrillos diarios. Hipercolesterolemia de reciente diagnóstico, manejada con dieta. Acude a la sala de urgencias
a las 6:00 AM, por dolor que inició cuatro horas antes al estar dormido, lo despertó; es opresivo, se localiza en región
retroesternal, de intensidad 10/10, irradiado a cuello y con sensación de “ahogo”. Acompañado de náusea, diaforesis profusa, fría
y sensación de evacuar el intestino. A la exploración física se encuentra diaforético, pálido +++, piel fría, FC 80 por minuto, TA
100/60 mmHg. Los campos pulmonares con estertores crepitantes bilaterales. EKG muestra ondas T invertidas y supradesnivel
del ST en derivaciones D-II, D-III y aVF.
1. De acuerdo con los cambios electrocardiográficos descritos, la localización de la lesión es:

a) Lateral alta

b) Anterior extensa

c) Septal

d) Lateral baja

e) Posteroinferior

2. Otro estudio auxiliar de diagnóstico que debe solicitarse son las enzimas; la de elección por su especificidad es:

a) DHL

b) CPK-MB

c) CPK total y fracción MM

d) TGP

e) TGO
3. El estudio que ayuda a establecer el diagnóstico de certeza de la afección coronaria es:

a) Angiografía coronaria

b) Gammagrama cardíaco

c) Electrocardiograma de 12 derivaciones

d) Enzimas cardíacas

e) Ecocardiografía Doppler

4. El tratamiento de elección al ingreso es:

a) Antiagregantes plaquetarios con ácido acetilsalicílico

b) Anticoagulante con heparina de bajo peso molecular

c) Anticoagulación con heparina convencional

d) Vasodilatador venoso y arterial con isosorbide

e) Trombolítico con estreptoquinasa o activador tisular del plasminógeno

5. En estos pacientes las complicaciones que se presentan con mayor frecuencia son:

a) Arritmias, bloqueos en la conducción e insuficiencia cardíaca derecha

b) Alteración a nivel de válvulas miocárdicas

c) Insuficiencia respiratoria aguda y SIRPA

d) Infartos cerebral y pulmonar

e) Fibrilación auricular y bloqueo de rama derecha

80 URGENCIAS
Masculino de 40 años de edad. Acude al servicio de urgencias consciente, con cefalea intensa, acúfenos, fosfenos y estado
nauseoso. Presenta ruidos cardíacos rítmicos con frecuencia cardíaca de 90 por minuto y presión arterial de 210/150 mmHg. Se
le diagnostica una crisis hipertensiva, por lo que es urgente reducir la presión arterial.
1. De los fármacos siguientes ¿cuál es el más indicado para lograr este efecto?

a) Hidroclorotiazida

b) Nitroprusiato de sodio

c) Captopril

d) Prazocín

e) Propranolol

2. El mecanismo de acción del fármaco indicado es:


a) Antagonismo sobre receptores adrenérgicos beta

b) Inhibición de la enzima convertidora de angiotensina

c) Bloqueo del simportador Na+-Cl- y aumento de la excreción de Na+ y H2O

d) Generación de óxido nítrico por biotransformación

e) Agonismo sobre receptores adrenérgicos alfa 2

3. El fármaco elegido puede provocar los siguientes efectos indeseables en el paciente:

a) Hipercalcemia y erupción cutánea

b) Hipotensión ortostática

c) Hipotensión y acumulación de tiocianato

d) Constipación y edema periférico

e) Arritmia y crisis de angina de pecho

4. Controlado el episodio agudo, usted decide prescribir el siguiente profármaco:

a) Prazocín

b) Losartán

c) Clortalidona

d) Propranolol

e) Enalapril

5. El efecto antihipertensivo de este último se atribuye a:

a) Bloqueo de los receptores adrenérgicos beta

b) Bloqueo de los receptores adrenérgicos alfa 1

c) Inhibición de la enzima convertidora de angiotensina

d) Inhibición del simportador Na+-Cl-

e) Antagonismo sobre receptores AT1 de angiotensina

81 URGENCIAS
Masculino de 62 años de edad. Tabaquismo intenso durante 30, sedentario, padece hipertensión arterial desde hace 10 años, con
tratamiento irregular no especificado. Tuvo pérdida súbita del estado de alerta hace aproximadamente 30 minutos. A la
exploración física, en coma, dificultad respiratoria, estertores a distancia, miosis bilateral, mirada conjugada a la izquierda,
pronación de extremidades superiores a los estímulos nociceptivos. La gasometría arterial: pH 7.3, PCO2 45, PO2 38, HCO3 29,
colesterol total 217 mg/dL.
1. Los datos clínicos orientan hacia daño en:

a) Corteza frontal

b) Corteza occipital

c) Ganglios basales

d) Protuberancia

e) Cerebelo

2. Los resultados de la gasometría indican:

a) Valores en límites normales

b) Acidosis metabólica

c) Acidosis respiratoria

d) Acidosis mixta

e) Alcalosis metabólica

3. El factor de riesgo más destacado para el problema neurológico en este caso es:

a) Hipertensión arterial

b) Dislipidemia

c) Tabaquismo

d) Sedentarismo

e) Obesidad

4. La probabilidad etiológica de acuerdo con la presentación súbita y sin cuadros previos semejantes sugiere:

a) Síndrome de Hakim

b) Hemorragia cerebral

c) Trombosis del seno longitudinal superior

d) Embolización por cardiopatía asintomática

e) Ateromatosis carotídea bilateral

5. De los siguientes fármacos, el que mejora el pronóstico neurológico del paciente es:

a) Dexametasona

b) Praziquantel
c) Manitol

d) Enoxaparina

e) Nimodipino

82 URGENCIAS
Femenina de cuatro años de edad. Traída por su madre por haber ingerido raticida hace tres horas, en cantidad no cuantificada,
seguida de náusea, vómito, dolor abdominal difuso y evacuaciones de consistencia disminuida en número de cuatro. A la
exploración física presenta tegumentos con buena coloración, deshidratación moderada, con residuos de polvo en manos, además
de dolor abdominal moderado a la palpación.
1. La acción inmediata que usted efectúa es:

a) Interrogatorio sobre la cantidad ingerida

b) Administración de hidrocortisona de 0.5 a 0.7 mg/kg/ dosis IV

c) Lavado gástrico con carbón activado o con solución inyectable

d) Inducción del vómito con aceite de ricino

e) Gel de aluminio y magnesio

2. Posteriormente, el manejo debe incluir:

a) Lavado de manos exhaustivo y estimulación de la diuresis

b) Lavado de manos exhaustivo, hidrocortisona a 0.5 mg/kg/dosis

c) Lavado de manos exhaustivo, aplicación de naloxona

d) Aplicación de antihistamínicos y enema evacuante

e) Carbón activado oral, hidrocortisona y antihistamínicos

3. Son manifestaciones neurológicas que puede presentar la niña, EXCEPTO:

a) Neuropatía periférica

b) Paresias y zonas de anestesia

c) Rigidez de nuca

d) Convulsiones

e) Sopor

4. Los siguientes son síntomas cutáneos que puede presentar la niña en días posteriores, EXCEPTO:

a) Lesiones hiperémicas con borde activo y pruriginosas

b) Lesiones eritematodescamativas
c) Alopecia

d) Bandas semilunares hipocrómicas sobre las lúnulas ungueales

e) Bandas de leuconiquia o de Mess

5. Las manifestaciones clínicas anteriores que puede presentar la paciente son secundarias a la intoxicación por:

a) Cloruros orgánicos

b) Clorados del etano

c) Dinitrofenoles

d) Bipiriano

e) Talio o zelio

83 URGENCIAS
Femenina de 39 años de edad. Acude a consulta y refiere que desde hace un mes presenta disminución del impulso vital e
irritabilidad importantes, hiporexia con pérdida de peso no cuantificada, dificultad para conciliar el sueño, alteración de la
memoria anterógrada. También manifiesta tristeza frecuente, así como desinterés por las actividades que le eran gratas y con las
que ya no experimenta placer.
1. El cuadro clínico mencionado pertenece al rubro de trastornos:

a) De adaptación

b) De ansiedad

c) De personalidad

d) Del estado de ánimo

e) Psicóticos

2. Entre los antecedentes hereditarios y familiares de primer grado, es significativo encontrar trastornos:

a) De personalidad

b) Bipolares

c) Neurológicos

d) Psicóticos

e) Por dependencia a sustancias

3. En los antecedentes personales no patológicos, deberá interrogarse exhaustivamente:

a) Los hábitos higiénicos

b) Los hábitos dietéticos


c) El uso de medicamentos y sustancias psicoactivas

d) La utilización del tiempo libre

e) La personalidad previa

4. Una vez descartadas condiciones médicas que produzcan esta sintomatología, el diagnóstico será:

a) Depresión atípica

b) Trastorno depresivo mayor

c) Trastorno distímico

d) Neurosis depresiva

e) Crisis de angustia

5. El tratamiento adecuado para esta entidad será con:

a) Ansiolíticos

b) Antidepresivos

c) Estabilizadores del ánimo

d) Programas recreativos

e) Psicoterapia únicamente

84 URGENCIAS
Masculino de 74 años, tabaquismo positivo, seis a ocho cigarros al día, alcoholismo ocasional sin llegar a la embriaguez. Se
conoce portador de diabetes mellitus tipo 2 e hipertensión arterial desde hace 15 años. Refiere que desde hace ocho años,
presenta episodios repetidos de dolor en hipogastrio y fosa ilíaca izquierda, el cual empeora después de comer y se alivia
transitoriamente después de defecar. Hace un año, presentó hemorragia rectal indolora, escasa, sin acudir al médico. Inició el
padecimiento actual hace dos días, con dolor abdominal súbito, persistente, localizado en el cuadrante inferior izquierdo,
hiporexia, estado nauseoso y escalofrío. Exploración física: temperatura 38.3°C, TA 140/95 mmHg, FC 96 por minuto, peso 94
kg, estatura 1.68 m. El abdomen se encuentra distendido, con resistencia muscular involuntaria, hipersensibilidad en el cuadrante
inferior izquierdo, se palpa masa fija y dolorosa además de ruidos intestinales disminuidos.
1. Con base en el cuadro clínico descrito, el diagnóstico más probable es:

a) Diverticulitis

b) Colitis ulcerosa

c) Colitis isquémica

d) Enfermedad de Crohn

e) Hemorroides internas

2. El estudio paraclínico más útil para confirmar el diagnóstico es:


a) Ultrasonografía

b) Proctosigmoidoscopía

c) Tomografía computarizada

d) Radiografía simple de abdomen

e) Estudio radiográfico con contraste de bario

3. El cuadro clínico presentado hace un año muy probablemente se debió a:

a) Angiodisplasia

b) Hemorroides mixtas

c) Hemorragia diverticular

d) Proctitis en enfermedad de Crohn

e) Actividad leve de colitis ulcerosa

4. El tratamiento indicado es:

a) Ayuno, líquidos IV y antibióticos

b) Corticoesteroides y sulfasalazina

c) Inyecciones esclerosantes o fotocoagulación

d) 6-mercaptopurina, sulfasalazina y metronidazol

e) Extirpación quirúrgica del segmento gangrenado

5. Son complicaciones de este padecimiento:

a) Pólipos adenomatosos y neumatosis cistoide del intestino

b) Absceso intraabdominal, fístulas y obstrucción intestinal

c) Prolapso rectal, necrosis cutánea y acrocordones anales

d) Amiloidosis y aumento en la capacidad litogénica de la bilis

e) Angiodisplasia cecal, isquemia fulminante y hemorragia intramural

Anda mungkin juga menyukai